{ "Tag": [], "Problem": "There are two people A and B. They can remove a coin at a time from a pile or split a pile into two non empty piles. The last person to remove the coin loses. The objective of the game is to lose. The game starts with A1 piles of single coins and A2 piles of two coins...How would the person lose based on the sequence (A1,A2,A3...)?\r\n\r\nI think this is the one about taking out a specific number that is bigger than the amount able to take out by one person but can be taken up if both people take the turn.\r\n\r\nI think it is similar to this:(A very basic game)\r\nA person can say 1,2 or 3numbers at a time. Two people keep saying the numbers until the person to say 21 loses. In this case, the second person always have to say up to the multiple of 4, which leads the person to victory.\r\n\r\nSo.. maybe there is a similar form of strategy to this game as well?", "Solution_1": "Amod lock this, the competition isnt over until 14th of october", "Solution_2": "Hey I read the downloads. I can't find which question is the same is this can you tell me?", "Solution_3": "lol its number 11 person A=michael, person B=robin. Your friend just changed a bit so that other people wouldn't notice. Its basically the same idea :ninja:\r\nIf someone to you the losing solution, then you friend could just write the exact opposite of the losing solution :roll: :wink:", "Solution_4": "Let:\r\n$n:$ The number of the piles\r\n$x:$ The total number of the coins\r\n$m:$ The number of the single coins\r\n\r\nIf $m$ is even and $x-n$ is also even then we will say that we have a [u]W-state[/u].\r\nElse we will say that we have an [u]L-state[/u].\r\n\r\n\r\nThe [b][u]objective[/u][/b] for each move is to reach a W-state.\r\n\r\n[color=brown]We will show that, every W-state can lead only to L-states.\nAlso, in every L-state, there is (at least one) move such that we can go to a W-state.[/color]\r\n\r\n\r\nFrom this point, when we say that $m$ changes, we will mean that its parity changes, that is, $m \\mod 2$ changes.\r\nWhen we say that $m$ keeps its value, we'll mean that $m \\mod 2$ keeps its value\r\n(for example, if the new value of $m$ is $m-2$, then we will say that $m$ doesn't change; it keeps its value)\r\n\r\n[hide=\"Let us first see the possible moves\"]\n$\\blacksquare$ Remove a coin from a pile with $k$ coins\n\nFor $x:$\nThe new value of $x$ is $x-1$. So $x$ [u]changes[/u]\n\nFor $n:$\nIf $k=1$ (if we remove from a pile with 1 coin) then, of course $n$ [u]changes[/u].\nElse it doesn't.\n\nIf $k=1$, then $m$ becomes $m-1$, so $m$ [u]changes[/u]\nIf $k=2$, then $m$ becomes $m+1$, so $m$ [u]changes[/u]\nIn any other case $m$ doesn't change.\n\nSummary:\n[quote]$x-n$ keeps its value if $k=1$\n$m$ keeps its value if $k>2$[/quote]\n\n\n$\\blacksquare$ Splitting\nWe split a pile of $k$ coins into two non-zero piles of $p$ and $q$ coins, such that $p+q=k$ (WLOG suppose that $p\\leq q$)\nOne should notice that $k$ must be at least 2.\n\nFor $n:$\nWe loose a pile but we obtain two new piles.\nSo $n$ becomes $n+1 \\Rightarrow n$ [u]changes[/u]\n\nFor $x:$\nOf course we have the same number of total coins, so $x$ [u]doesn't change[/u]\n\nFor $m:$\nWe saw that $k\\geq 2$. So we don't [u]loose[/u] any of the single coins by splitting. \nWe could only [u]obtain[/u] more single coins, depending on the values of $p,q$\n \nIf $1
1$, as we saw, $x-n$ changes, so we go to an L-state\nIf $k=1$, then $x-n$ keeps its value, but $m$ changes. That is an L-state.\n\nConclusion\nThis means that, if we \"give\" to the opponent a W-state, then he can't give us a W-state. \nHe'll give to us an L-state. \nOur goal is to convert an L-state to a W-state. Let's see how:\n------------------------------------------------------------------------[/hide]\n[hide=\"Now we will see that, if the state is L, then we can find a move, such that the result is a W-state\"]\nL-state means that at least one of $m,x-n$ is odd.\n\n[color=brown]Note\nIf $x-n$ is odd, then there is at least one pile with $k>1.$\nIndeed, if we have only single coins, then $x=n\\Rightarrow x-n$ is even[/color]\n\nWe'll check 3 cases:\n\n[hide=\"a) m odd and (x-n) even\"]\nSince $m$ is odd, there is at least one single coin.\nWe will remove this coin.\nRemember the rules of removing:\n[quote]$x-n$ keeps its value if $k=1$\n$m$ keeps its value if $k>2$[/quote]\nHere, $k=1$, so $x-n$ remains even and $m$ becomes even\nSo we have a W-state\n------------------------------------------------------------------------[/hide]\n[hide=\"b) m odd and (x-n) odd\"]\nWe have the sub-cases:\n\n$b.1)$ If there exists a pile with $k>2$ coins\nWe split this pile into two piles of $1$ and $k-1$ coins respectively.\nRemember the rules of splitting:\n[quote]$x-n$ changes\n$m$ changes if [u]just one[/u] of the new piles is a single coin[/quote] \n\nSo $x-n$ changes and $m$ changes $\\Rightarrow x-n \\equiv m \\equiv 0 \\mod 2$\n\n$b.2)$ If there is not a pile with $k>2$, then we can be sure that there exists at least one pile with $k=2$ coins (see note above)\nWe remove one coin from this pile\nRemember the rules of removing:\n[quote]$x-n$ keeps its value if $k=1$\n$m$ keeps its value if $k>2$[/quote]\n\nHere, it is $k=2$, so $x-n$ changes and $m$ changes $\\Rightarrow x-n \\equiv m \\equiv 0 \\mod 2$\n------------------------------------------------------------------------[/hide]\n[hide=\"c) m even and (x-n) odd\"]\nWe have the sub-cases:\n\n$c.1)$ If there is some pile with $k=2$ coins\nThen we split this pile\nRemember the rules of splitting:\n[quote]$x-n$ changes\n$m$ changes if [u]just one[/u] of the new piles is a single coin [/quote]\n\nSo $x-n$ changes and $m$ does not $\\Rightarrow x-n \\equiv m \\equiv 0 \\mod 2$\n\n\nc.2) If there is not a pile with $k=2$ coins\nThen we can be sure that there is some pile with $k>2$ coins (see note above)\nWe remove one coin from this pile\n\nRemember the rules of removing:\n[quote]$x-n$ keeps its value if $k=1$\n$m$ keeps its value if $k>2$[/quote]\n\nHere, it is $k>2$, so $x-n$ changes and $m$ doesn't $\\Rightarrow x-n \\equiv m \\equiv 0 \\mod 2$\n------------------------------------------------------------------------[/hide]\n----------------------------------------------------------------------------------------------------------\n[/hide]\r\n\r\n[size=134][color=blue]Conclusion[/color][/size]\r\nSo, we can always convert a W-state to an L-state\r\n\r\nIf the two players know this strategy, then the initial state will show who will be the winner.\r\n\r\nIf it is a W-state, then the first player can't convert it to a W-state, so he will loose\r\nIf it is an L-state, then the first player can convert it to a W-state, so he'll win",
"Solution_5": "Nice proof pontios.\r\nI came up with a similar solution.\r\nI did\r\nLet A=pile of coins\r\n B=number of piles\r\n 0=even\r\nIf it is even(A)xodd(B)+odd(A)xeven(B)+even(A)xodd(B)+...\r\nor odd(A)xeven(B)+even(A)xodd(B)+odd(A)xeven(B)+...\r\nDuring the person B's turn; It is always a win.\r\nI have found this works up to any number of pile of coins up to 4. The only catch was that If person A had 1odd left he would lose. I was wondering would this strategy work? And up to what extent? Could someone contradict/point out/help me please?\r\nThanks.\r\n\r\nPS:I have concluded that pontios's solution is the most correct so far. However, is there any shorter solution?",
"Solution_6": "Actually, when I posted my solution I couldn't simplify it more. And the proof was indeed hard. Thank you for revising this topic, because I had the chance to try it more. I wrote a script which calculates all the possible states starting from (1,2,3,4,5). \r\nFinally, I found a simpler solution to this problem.\r\n\r\nHowever, I can't understand some points in your post\r\n\r\n[quote=\"max1546\"]Let A=pile of coins\n B=number of piles\n 0=even\n[/quote]\nwhat is A?\n\n[quote=\"max1546\"]If it is even(A)xodd(B)+odd(A)xeven(B)+even(A)xodd(B)+...\nor odd(A)xeven(B)+even(A)xodd(B)+odd(A)xeven(B)+...\n[/quote]\r\n\r\nwhat does this sentence mean? :huh:",
"Solution_7": "Sorry Pontios for being so unclear, I was rushing it.LOL\r\nhmm I'll try to explain some of the things you asked.\r\nRemember though, this MIGHT make you more confused.\r\nSo i'll do my best.\r\n\r\nIf there is 3coins stacked on top of each other, B=3, A=1.\r\nIf there are 6coins stacked so that it is 3coins per pile, it is B=3, A=2.\r\n\r\nThe other part was, I tried some trial and error and got the answer that if it is the person's turn and he starts off with;\r\n1(A)x1(B)=Certain chance of winning.\r\n1(A)x2(B)=Win\r\n2(A)x1(B)=Lose\r\n1(A)x3(B)=Win\r\n1(A)x4(B)=Lose\r\n1(A)x5(B)=Win\r\n2(A)x2(B)=Win\r\nAnd so on.I tried up to 4(A) and my sequence given worked. I was trying to take in to account every single move either could make and if the person who starts has a chance of winning no matter what the other person does, he wins. On the other hand, if the other person could win with at least one move no matter what the starting person does, I counted it as a loss.\r\n\r\nAhh, even I'm getting slightly confused. I hope this explained slightly more. Still I think your solution is the best so far. If I have time, I'll try and improve my solution and post it again. Hopefully it will be easier/clearer.\r\n\r\nThanks",
"Solution_8": "Let's re-formulate the problem\r\n[quote]There are two people A and B. They can remove a coin at a time from a pile or split a pile into two non empty piles. \nThe last person to remove the coin [u][color=red]wins[/color][/u]. \n\nThe game starts with \nA1 piles of single coins \nA2 piles of two coins...\nA3 piles of three coins...\nHow would the person [u][color=red]win[/color][/u] based on the sequence (A1,A2,A3...)?\n[/quote]\r\n\r\n[hide=\"My new solution\"]\nEvery state can be represented by the sequence $A_{1}A_{2}A_{3}A_{4}...$\n\n\nLet $\\boxed{x = A_{1}\\mod 2}$ and $\\boxed{y=(\\sum{ A_{2k}})\\mod 2}$ \n\n(we don't need the numbers A3,A5,A7,...)\n\nEquivalently, we can say that we take the last binary digit of $A_{1}$ and $\\sum{ A_{2k}}$\n\n\n\nNext, we calculate the value $S=x \\vee y$ from the folowing table:\n\n$\\begin{array}{|c|c|c|}\\hline x & y & x \\vee y \\\\ \\hline 0 & 0 & 0 \\\\ \\hline 0 & 1 & 1 \\\\ \\hline 1 & 0 & 1 \\\\ \\hline 1 & 1 & 1 \\\\ \\hline \\end{array}$ (Boolean OR)\n\nAnd we are finished!\n\nThe state $(A_{1}A_{2}A_{3}A_{4}...)$is marked with the label $S$. \nWe will say that we have an [b]S[/b]-state\n\n\nIf you start playing from a [b]1[/b]-state, then you will [u]definitely[/u] win \nIf you start playing from a [b]0[/b]-state, then you will [u]definitely[/u] loose \n\nSo, after each move we should take care such that the result is a [b]0[/b]-state.\n\n[color=darkred]Note\n - The state $(A_{1}A_{2}A_{3}A_{4}...)=(1000...)$ is clearly a [b]1[/b]-state (if you start from this state, you'll win)\n - The state $(A_{1}A_{2}A_{3}A_{4}...)=(0000...)$ can be defined as a [b]0[/b]-state[/color][/hide]",
"Solution_9": "[hide=\"Proof\"]\nAgain the proof is a litle complicated but no so much as the previous\n[hide=\"opponent s moves\"]\nThe opponent plays always starting from a [b]0[/b]-state, that is, $(x,y)=(0,0)$\n\nThat is, he has \n - an even number of single coins, and\n - an even number of even piles\n\n[i][u]Even pile[/u]: a pile consisting of 2 or 4 or 6, ... coins[/i]\n\nCan he make a move to go to a [b]0[/b]-state?\nThe answer is [b][u]no[/u][/b]\n\nLet's see the possible opponent's moves:\n\n$\\hline$\n[color=green][size=150]Removing[/size]\nRemove 1 single coin . Bad move! $x$ becomes $1$\nRemove 1 coin from a pile with $2$ coins. Bad move! $x,y$ become $1$\nRemove 1 coin from a pile with $m\\geq 3$ coins. Then we have:\n$A_{m}$ decreases by $1$\n$A_{m-1}$ increases by $1$\n\nSome of $m,m-1$ is even, so $y$ becomes $1$[/color]\n$\\hline$\n\n[color=brown][size=150]Splitting[/size]\nSplit the pile $A_{m+n}$ into two nonzero piles $A_{m},A_{n}$ (suppose $m\\geq n$)\n\nIf $m=n=1$ then he obtains two single coins ($x$ doesn't change) but he destroys one double-coin pile (so $y$ becomes 1)\n\nIf $m>n=1$ then he obtains 1 single coin ($x$ becomes 1)\n\nIf $n\\geq 2:$\n\nIf $m+n$ was odd, then [u]exactly one[/u] of $m,n$ is even (Bad move; $y$ becomes 1)\n\nIf $m+n$ was even:\n\nEither $m,n$ are both even\nOr they are both odd\n\nAt either case he destroys the y-balance[/color]\n\n\nSo the opponent can't turn the game to [b]0[/b]-state\n$\\hline$[/hide]\n[hide=\"Your moves\"]You start from a [b]1[/b]-state. Let's see if there is a way to turn the game to a [b]0[/b]-state\n\nIn the following, $m$ will represent a natural number $m \\in \\{0,1,2,...\\}$\nThere are three cases such that a state is a [b]1[/b]-state:\n\n$(x,y) = (1,0)$\n$(x,y) = (0,1)$\n$(x,y) = (1,1)$\n\n[hide=\"(1,0)\"]There is an odd number of single coins. You want to make it even.\n[color=red][b]Solution:[/b] Just remove a single coin[/color] ($x$ becomes 0)\n$\\hline$[/hide][hide=\"(0,1)\"]\nYou have an odd number of even piles.\n$\\blacktriangleright$ If there is a pile with $m+3$ coins \n[color=red][b]Solution: [/b] remove a coin from it.[/color]\nYou obtain a pile with $m+2$ coins, but you loose the initial pile with $m+3$ coins\nExactly one of $m+3$ and $m+2$ is even, so you adjust the y-balance ($y$ becomes 0)\n\n$\\blacktriangleright$ If there is not a pile with $m+3$ coins. Then you have only double coins (don't care about the single coins)\n[color=red][b]Solution: [/b]Split one double-coin pile into two single coins.[/color] So you adjust the y-balance\n$\\hline$[/hide][hide=\"(1,1)\"]\nYou have an odd number of single coins and an odd number of even piles\n\n$\\blacktriangleright$ If there is a pile with $m+3$ coins, then \n[color=red][b]Solution: [/b]split 1 coin[/color]\nYou obtain a single coin so $x$ becomes 0\nYou obtain a pile with $m+2$ coins, but you loose the initial pile with $m+3$ coins. For the same reasons as above, you adjust the value of $y$\n\n$\\blacktriangleright$ If there is not a pile with $m+3$ coins. Then you have only double coins (and some odd single coins)\n[color=red][b]Solution: [/b]Just remove one coin from a double-coin pile.[/color]\nThe double coin becomes single.\nSo you obtain one single and loose one double, as a result both the values of $x$ and $y$ are fixed\n$\\hline$[/hide]\n\nYou can see the diagram:\n\n$(x,y)=\\{\\begin{array}{{l}(1,0) \\Rightarrow \\text{Remove a single coin}} \\\\ \\\\ (0,1) \\Rightarrow \\exists m>0 : A_{m+3}>0 \\{\\begin{array}{l}\\exists \\Rightarrow \\text{Remove a coin from }A_{m+3}\\\\ \\not\\exists\\Rightarrow \\text{Split a coin from }A_{2}\\end{array} \\\\ \\\\ (1,1) \\Rightarrow \\exists m>0 : A_{m+3}>0 \\{\\begin{array}{l}\\exists \\Rightarrow \\text{Split a coin from }A_{m+3}\\\\ \\not\\exists\\Rightarrow \\text{Remove a coin from }A_{2}\\end{array} \\end{array}$\n\n\n\n\n\n[/hide][/hide]"
}
{
"Tag": [
"AoPSwiki",
"Asymptote",
"vector",
"LaTeX",
"AMC 8"
],
"Problem": "The AoPSWiki Words of the Week for Nov 8-14 are [[Asymptote]] (the graph concept, not the vector graphics language used with LaTeX) and [[AMC historical results]].",
"Solution_1": "Shouldn't each diff be posted by the admins?",
"Solution_2": "By \"AMC Historical Results\", do you mean perfect scores on AMC 8, or perfect scores on the AMC 12, or participating in IMO when you were 10 years old?",
"Solution_3": "[quote=\"1=2\"]By \"AMC Historical Results\", do you mean perfect scores on AMC 8, or perfect scores on the AMC 12, or participating in IMO when you were 10 years old?[/quote]The community should decide these issues on the article's discussion page. I started off the discussion but the page should be whatever people think is most relevant."
}
{
"Tag": [],
"Problem": "I know I can solve all of them and I'm more than happy to explain (I write my explanation on computers) to you. I might be able to PM you...",
"Solution_1": "i lived in North Carolina too!\r\n\r\nIn Charlotte, Mecklenburg district"
}
{
"Tag": [
"AMC",
"USA(J)MO",
"USAMO"
],
"Problem": "hi guys, i'm gr 12 this year, so everything for math is over for me, but i just want to know how u guys handle this so i can kick myself for this.\r\n\r\nok, i love math, i've done all of the waterloo books and more\r\nwhen i was preparing for comc this year, every contest i can get around 65, but actual thing i got 52 \r\nsame for euclid 2003, every practice i can get 80+ but i got 55 in 2003\r\nthis year, i think i can only get 7*\r\nboth from being too nervous\r\nhow do u guys get rid of nervousness so that u can do ur best?\r\nplus what materials do u guys work on?",
"Solution_1": "i know i can do a lot better, because last year, i got 32 on hypatia, which is a 83rd in canada\r\ni know it is not very good compare to some of u guys, but it's good for me\r\nbtw, i got 111 on fermat because of nervous, and retarded mistakes because i was expecting 130+ because that's what i've been getting on my practice contests\r\n\r\nlong story short, i got my butt wooped because i was nervous, any ideas how to get past this? (other than, do more questions)",
"Solution_2": "who cares... i was nervous too",
"Solution_3": "um, comments like yours don't help, bugzpodder.\r\n\r\nhe wants someone to give advice. you shouldn't have said anything if you didn't have any.",
"Solution_4": "sorry i was trying to say everyone is nervous... it didnt came out right",
"Solution_5": "It gets better after you participate in more contests (I don't mean just practice, rather actually sitting in the test room).\r\n\r\nAt first, I can barely handle the stress of a 30 min contest. After a while, you just get used to the idea of math competitions. Now, sitting 4.5h for say USAMO isn't a problem for me.\r\n\r\nAs well, after you do more problems (especially the proving ones), you get a sense of what solutions are right and what are not.\r\n\r\nThen again, of the two COMCs that I've done, I never quite figured out where I lost the marks. The Olympiads are definitely better, at least for me. For both USAMO and APMO, I got marks divisible by 7 :).\r\n\r\nLastly, one of my philosophies for math contests is that, one's performance on a contest is no greater than 33% of his/her actual ability.",
"Solution_6": "Remember it is just you and the problem. It is an icky little thing, but just slay it, take your time as you do with the practices. If you are getting panicky, do something else for a little while. I sometimes go for a walk to the toilet and back. Or eat something. Or take a few deep breaths. Feel your mind with thoughts of the problem, and not with thoughts like \"I must solve this to get a silver\".... concentrate, basically.",
"Solution_7": ":D :D :D :D"
}
{
"Tag": [
"probability",
"expected value"
],
"Problem": "A certain coin is weighted such that the chance of flipping heads is $ \\frac{1}{3}$ and the chance of flipping tails is $ \\frac{2}{3}$. Suppose that we win $ \\$3$ if we flip a heads on a coin toss, but lose $ \\$2$ if we flip tails. What is the expected value, in dollars, of our winnings after one flip? Express your answer as a common fraction.",
"Solution_1": "By definition, this is $ 3\\cdot \\frac{1}{3}\\plus{}(\\minus{}2)\\cdot \\frac{2}{3} \\equal{}\\boxed{\\frac{\\minus{}1}{3}}$"
}
{
"Tag": [
"analytic geometry",
"inequalities",
"calculus",
"derivative",
"function"
],
"Problem": "If $a,b,c \\in [0,\\infty),bc\\ne 0 ,$ show that \\[1 \\le \\frac{a+b+c}{\\sqrt{a^{2}+b^{2}+c^{2}}}\\le \\sqrt{3}\\; .\\; \\; \\; \\; (\\mbox{corrected !})\\] [size=75][b]Try to generalize these inequalities.[/b][/size]",
"Solution_1": "I think it generalized should be something like this:\r\n$1\\leq\\frac{a_{1}+a_{2}+...+a_{n}}{\\sqrt{a_{1}^{2}+a_{2}^{2}+...+a_{n}^{2}}}\\leq\\sqrt n$\r\nGiven that they are all >=0 and at least one is non-zero.\r\n\r\nTo prove this:\r\nFirst, note that scaling all the variables by some factor k leads to the same equation since the k cancels in the numerator and denominator. This means that we can restrict our interval to some finite interval, because anything outside that can be achieved by scaling. Take all the variables to be in the interval [0,1].\r\n\r\nNow I want to say that you only need to consider the endpoints, that is each variable=0 or 1, but I'm not sure why that would be allowed. If that works, then you can see that the minimum is 1 and the maximum is $\\sqrt n$.\r\n\r\nAnother way to prove the minimum:\r\nTake all the variables to be coordinates in n-dimensional space. Then the minimum path from the origin to that point is the denominator. The numerator is an alternative path. That means the numerator>=denominator, and equality gives when it =1.",
"Solution_2": "For [b]Wumbate[/b]'s generalization, RMS-AM gives the RHS: \\[\\frac{a_{1}+\\ldots+a_{n}}{n}\\le\\sqrt{\\frac{a_{1}^{2}+\\ldots+a_{n}^{2}}{n}},\\] \\[\\frac{a_{1}+\\ldots+a_{n}}{\\sqrt{a_{1}^{2}+\\ldots+a_{n}^{2}}}\\le\\sqrt{n}.\\] Since all the $a_{i}$ are non-negative, then \\[a_{1}^{2}+\\ldots+a_{n}^{2}\\le (a_{1}+\\ldots+a_{n})^{2}.\\] Hence, \\[1\\le\\frac{a_{1}+\\ldots+a_{n}}{\\sqrt{a_{1}^{2}+\\ldots+a_{n}^{2}}},\\] giving the LHS.",
"Solution_3": "Using Cauchy in Engel form,\r\n$\\frac{a_{1}^{2}}{1}+\\frac{a_{2}^{2}}{1}+\\cdots+\\frac{a_{n}^{2}}{1}\\geq \\frac{{(a_{1}+a_{2}+\\cdots+a_{n})}^{2}}{n}$\r\n$\\Rightarrow \\frac{a_{1}+a_{2}+\\cdots+a_{n}}{\\sqrt{a_{1}^{2}+a_{2}^{2}+\\cdots+a_{n}^{2}}}\\leq \\sqrt{n}$",
"Solution_4": "Actually, the inequality cannot be strengthened past $1 \\le \\frac{a_{1}+a_{2}+...+a_{n}}{\\sqrt{a_{1}^{2}+a_{2}^{2}+...+a_{n}^{2}}}\\le \\sqrt{n}$, because setting $a_{1}=a_{2}=...=a_{n-1}=0$ yields a value of 1 for the expression. It is easy to prove the RHS using Cauchy, and the LHS follows as long as the terms are nonnegative.",
"Solution_5": "[quote=\"K81o7\"]Actually, the inequality cannot be strengthened past $1 \\le \\frac{a_{1}+a_{2}+...+a_{n}}{\\sqrt{a_{1}^{2}+a_{2}^{2}+...+a_{n}^{2}}}\\le \\sqrt{n}$, because setting $a_{1}=a_{2}=...=a_{n-1}=0$ yields a value of 1 for the expression. ........[/quote]\r\nThanks for interest,",
"Solution_6": "Even if $bc=0$, you can let $a=0$, and take the limit of the expression as $b \\to 0$ (with $c \\neq 0$). That value is the same: ie, by continuity, you can always make the expression as close to 1 as you want it to be.",
"Solution_7": "[quote]... following problem arise: [b]prove or disprove that [/b]...... FALSE ![/quote]\r\nSorry, thanks for interest,Alex",
"Solution_8": "Even if b=c>0, then you have this:\r\n$\\frac{a+2b}{\\sqrt{a^{2}+2b^{2}}}$\r\nNow take a->infinity and/or b->0 and this tends to 1. So it would still have 1 as a lower bound.\r\n\r\n\r\n\r\nFor the general inequality, I still want to know if there is a way to reduce the \"checking\" to values of 0 or 1.\r\n\r\nI think it would be nice if you could reduce all the variables to only 0 or 1 - it's easy, it happens to work, but I'm not sure with the reasoning.\r\n\r\nIf you take a partial derivative of the function with respect to one of the variables, and set that to 0, you will find the max and mins. Now if you assume all the other variables are 0 or 1, then the only interesting value of that variable is 0 or 1.\r\n\r\nI'm not too good with multivariable stuff, so if someone else could explain, that would be nice."
}
{
"Tag": [],
"Problem": "When are the student solutions for round 4 going to be posted? I'm curious to see some of the commended techniques for solving this round's problems.",
"Solution_1": "Probably next week.",
"Solution_2": "They are now posted."
}
{
"Tag": [
"linear algebra",
"matrix",
"vector",
"probability and stats"
],
"Problem": "hi I 'm looking for informations about this kind of regression.\r\n\r\nWe try to estimate a symetric $ m$x$ m$ matrix, $ A$ for the following regression model : \r\n\r\n\\[ y_ty_t^T\\minus{}I_m\\equal{}A(x_{t}x_{t}^T\\minus{}I_m)A\\plus{}E_t\\] \r\n\r\nwith $ y_t,x_t$ two observations vectors of dimension $ m$, and $ E_t$ is a symetric matrix representing model errors.\r\n\r\nTo estimate this, $ A$ is calcutlated by minimising the the average Fr\u00f6benius norm of $ E_t$ matrices over the set of $ A$ in the space of symetric matrices. \r\n\r\nThis method is apparently known as a \"non linear least square\" method. I have tried to find some information about this kind of regression and estimation method in this particuliar context but i couldn't find anything. Basically I have found this strange regression in a statistical article where it is presented as \"classic\" but as I don't have a very large culture in statistics I didn't got much success on my googling research. \r\n\r\nIf anyone could lead me to some references or web links I would appreciate \r\n\r\nbye",
"Solution_1": "If it was presented in a research article then your best bet is to look at the references. I don't mean to state the obvious but if it is listed as \"classic\" then several other papers in the references should have information pertaining to it."
}
{
"Tag": [
"calculus",
"derivative",
"algebra",
"polynomial",
"induction",
"real analysis",
"real analysis solved"
],
"Problem": "i dont know whether the problem is simple or not.It looks very harmless but I cant solve it. \r\nFind the nth derivative of e^(-x 2 ) \r\n\r\nLooks like I am getting stupid day by day. \r\nPlease hurry I want the solution desperately.\r\n\r\nI can feel that there will be some reduction or recursive type of formula but I want the result to involve only n,e,x.",
"Solution_1": "Do you know [b]Hermite[/b] polynomials?",
"Solution_2": "$(e^{-x^2})^{(n)}=(-1)^nH_n(x)e^{-x^2}$ with \r\n\r\n$H_0(x)=1$\r\n$H_1(x)=2x$\r\n$H_2(x)=4x^2-2$\r\n\r\nBy induction \r\n\r\n$H_{n+1}(x)=2xH_n(x)-2nH_{n-1}(x)$",
"Solution_3": "f=e^(-x^2)\r\nf'=-2xe^(-x^2)\r\nf''=(-2x)^2 e^(-x^2)\r\nf'''=(-2x)^3 e^(-x^2)\r\n\r\nTherefore\r\n\r\nf^(n)=(-2x)^n e^(-x^2)",
"Solution_4": "Product rule...",
"Solution_5": "Just heard about them.I will surely be glad if u tell me more about that.\r\nI know how to solve recursions & generating functions.\r\nThats it.",
"Solution_6": "Sorry but I didnt understand what Jayscholar has tried to say\r\nIs it really that simple!",
"Solution_7": "[quote=\"the game\"]Sorry but I didnt understand what Jayscholar has tried to say\nIs it really that simple![/quote]\r\nNo, it isn't that simple. Jaysscholar is simply wrong, which is what jmerry was saying in #5.",
"Solution_8": "[quote=\"jmerry\"]Product rule...[/quote]\r\nwhich states that $ (fg)' = f'g+fg'$",
"Solution_9": "Certainly I know product rule.How will the knowledge of product rule help the solution of Hermite polynomials.I presume it will be solved by generating functions.",
"Solution_10": "Thought it to be a lot easier than that.At least the recursion relation looked simple.Thanks any way."
}
{
"Tag": [
"number theory",
"greatest common divisor",
"modular arithmetic",
"relatively prime",
"Divisibility Theory"
],
"Problem": "If $n$ is a natural number, prove that the number $(n+1)(n+2)\\cdots(n+10)$ is not a perfect square.",
"Solution_1": "The largest power of $ 2$ that divides $ \\frac{(n\\plus{}10)!}{n!}$ is $ 32$, so it cannot be a perfect square.",
"Solution_2": "That doesn't quite work ($ n \\equal{} 54$, for example).\r\n\r\n\r\nSuppose the product is a perfect square. Among the ten terms in the product:\r\n\r\n- five are even\r\n- at most two are odd and divisible by 3\r\n- at most one is odd and divisible by 5\r\n- at most one is odd and divisible by 7\r\n\r\nSo at least one term is not divisible by any of 2,3,5,7. This term is relatively prime with the others since any common divisor of two terms is at most 9. So this term is a perfect square, and at least $ 11^{2}$.\r\n\r\nNow note that this is the only square in our product, otherwise we would have a square greater than or equal to $ 11^{2}$ differing from another square by at most 9.\r\n\r\nIn particular, there must be exactly one term not divisble by any of 2,3,5,7. So we must have equality in all of our counts, and no odd term can be divisible by 15, 21, or 35.\r\n\r\nConsider the odd terms divisible by 3. Neither are divisible by 2,5,7, and the gcd between either of these terms and any another one is at most 9, therefore a power of 3.\r\n\r\nSo we can factor out the largest power of 3 from each of these terms. Each quotient that results is relatively prime to all other terms and are both squares. So it follows that each odd term divisible by 3 is either a square or three times a square.\r\n\r\nWe know from above that neither can be squares, so both are three times a square. But this also produces two squares that are too close (in particular, they differ by 2). This is a contradiction, so the product cannot be a square.",
"Solution_3": "We can prove it by using problem A9 in [url=http://www.mathlinks.ro/Forum/viewtopic.php?t=150377]here[/url].\r\n\r\nFrom ten consecutive numbers we have one which is relatively prime to others. Remove this number from the list we have nine numbers which we can chose five from them which are consecutive. For five consecutive we will prove that there's one prime to others else. Indeed, there are at most two ones which is divisible by $ 3$, and if there're two ones then one is odd and one is even. In case we remove one which is even, so one left there're at most two even numbers from $ 4$ number left or $ 3$ for the left case. Remove them we have one odd left or two odd but in which there's one is not divisible by $ 3$. So there's one number which is odd, not divisible by $ 3$. This number from list $ 5$ consecutive numbers is relatively prime to others else. Remove it we get four numbers which always find two are consecutive, and then of cause relative each other. So we could find three numbers which is relatively to any $ 9$ numbers else. Now note that if $ a$ is square then $ a\\plus{}1,a\\plus{}2$ are not and $ a\\minus{}1,a\\minus{}2$ are not of $ a>2$. So in ten consecutive numbers it could not contains more than two squares. This proves the problem.",
"Solution_4": "[quote=\"pluricomplex\"]\nSo we could find three numbers which is relatively to any $ 9$ numbers else. [/quote]\r\n\r\nthis isn't true. for example, take n=209, then only 211 is relatively prime to other nine numbers because 210 212 214 216 218 are even numbers;213 216 219 are multiple of 3; 210 and 215 are multiple of 5; 210 and 217 are multiple of 7\r\n\r\nthe mistake is that you may choose a number in the five consecutive numbers which is relatively prime to other $ 4$ numbers, but it maybe not relatively prime to other $ 9$ numbers.",
"Solution_5": "[quote=\"Peter\"]If $ n$ is a natural number, prove that the number $ (n \\plus{} 1)(n \\plus{} 2)\\cdots(n \\plus{} 10)$ is not a perfect square.[/quote]\r\n\r\nThere is a much stronger result proven by Erdos, \"The product of consecutive integers is never a power\" you can check it at:\r\n[url]http://www2.renyi.hu/~p_erdos/1975-46.pdf[/url]",
"Solution_6": "There was such a problem at Latvian Open Olympiad in Mathematics:\r\n\r\nIt is given that $ n$ is a natural even number. Let $ R \\equal{} n\\left(n\\plus{}1\\right)\\left(n\\plus{}2\\right)\\left(n\\plus{}3\\right)$.\r\n[list][i]a[/i]) Can $ R$ be a square of a natural number?[/list][list][i]b[/i]) Can $ R$ be a qube of a natural number?[/list]",
"Solution_7": "I think all your solutions don't work, why?\nWe note that $n \\equiv 1 \\pmod{8},n \\equiv 1 \\pmod{9},n \\equiv 1 \\pmod{25}$\n$n \\equiv 43 \\pmod{49},n \\equiv 111 \\pmod{11^2}$, \nBy Chinese remainder theorem, there always exists $n$ satisfies all conditions above, then $11^2.2^8.3^4.7^2.5^2||(n+1)(n+2)...(n+10)$ so $mod(11,2,3,7,5)$ don't work!, In that cases, we only can consider the modular of prime number which is large enough but It's impossible",
"Solution_8": "[quote=\"mszew\"][quote=\"Peter\"]If $ n$ is a natural number, prove that the number $ (n \\plus{} 1)(n \\plus{} 2)\\cdots(n \\plus{} 10)$ is not a perfect square.[/quote]\n\nThere is a much stronger result proven by Erdos, \"The product of consecutive integers is never a power\" you can check it at:\n[url]http://www2.renyi.hu/~p_erdos/1975-46.pdf[/url][/quote]\n\nThe URL cannot use anymore.\nI found a new one [url]http://www.renyi.hu/~p_erdos/1975-46.pdf[/url]",
"Solution_9": "just consider divisibility by all primes less then $10$ and you'll face contradiction"
}
{
"Tag": [],
"Problem": "[color=purple]Trang chu\u0309 cu\u0309a ML co\u0301 ph\u00e2\u0300n Problem of day anh Tu\u00e2n co\u0301 th\u00ea\u0309 cho em bi\u00ea\u0301t \u0111i\u00ea\u0300u ki\u00ea\u0323n \u0111\u00ea\u0309 ba\u0300i toa\u0301n cu\u0309a mi\u0300nh \u0111\u01b0\u01a1\u0323c cho\u0323n ko a\u0323 :) \n Thanks s\u00ea\u0301p :D [/color]",
"Solution_1": "T\u00f4i c\u0169ng mu\u1ed1n h\u1ecfi v\u1ea5n \u0111\u1ec1 n\u00e0y. Nh\u01b0ng t\u00f4i ngh\u0129 mu\u1ed1n \u0111\u01b0a b\u00e0i l\u00ean l\u00e0 do c\u00e1c master l\u00e0m."
}
{
"Tag": [
"conics",
"geometry proposed",
"geometry"
],
"Problem": "Is it possible to find three conics in the plane such that any straight line in the plane intersects at least two of the conics and through any point of the plane pass tangents to at least two of them?",
"Solution_1": "Problem is too ambiguous..."
}
{
"Tag": [
"topology",
"function",
"real analysis",
"real analysis solved"
],
"Problem": "could prove that $\\mathbb{R}^{\\mathbb{R}}$ is seprable or disprove it .it is not that hard that $\\mathbb{R}^{P(P(\\mathbb{R}))}$ is not seprable with product topology.",
"Solution_1": "What is product topology?",
"Solution_2": "Lemma: The cardinality of a separable space is at most that of the real numbers.\r\nProof: Take a countable dense subset. Every element of the space can be expressed as the limit of a sequence in that set. There are as many sequences as there are real numbers.\r\n\r\nNow, $\\mathbb{R}^{\\mathbb{R}}$ has larger cardinality than $\\mathbb{R}$, so it cannot be separable.",
"Solution_3": "[quote=\"jmerry\"] Every element of the space can be expressed as the limit of a sequence in that set. [/quote]\r\nI would really love to see your proof of that, jmerry. It is not obvious at all if the base of topology at a point is not countable...",
"Solution_4": "My previous post assumed a metric topology.",
"Solution_5": "Yeah, I thought so too. The problem is that $\\mathbb R^{\\mathbb R}$ with the product topology is not metrizable...",
"Solution_6": "Yes, I think it's not even first-countable. :?",
"Solution_7": "yes not being first countable is almost obvious you can take these points :all with finite zero and the other index is 1 then the point that has all zero in all index is limit point of these sequence but you can not find the sequense such that its limit is all zero in all index so it is not first countable .",
"Solution_8": "If the product topology is weak i.e the basis is of the form $\\Pi{ \\{U_{\\alpha}\\}}_{\\alpha \\in \\mathbb{R}}$ where all but finite $U_{\\alpha}$ are the whole $\\mathbb{R}$ and the others are open sets in $\\mathbb{R}$, then I think we can show that the space is seperable as follow :\r\n Let $I$ be the set of all horizontal intervals in $\\mathbb{R}^2$ whose endpoints have rational coordinates. This set is countable. Let $Q$ be the set of function $f$ ( and there fore $Q$ is a subset of $\\mathbb{R}^{\\mathbb{R}}$) whose graph deleted the points $(x,f(x)), f(x)=0$ is consisted of finite number of intervals in $I$. $Q$ is also countable. We will prove that $Q$ is densed in $\\mathbb{R}^{\\mathbb{R}}$.\r\n Now let $\\Pi {\\{{U_{\\alpha}\\}}_{\\alpha \\in \\mathbb{R}}}$ be an open set in the space as above.\r\nCall those open sets $U_{\\alpha}$ which are not the whole $\\mathbb{R}$, $U_1,\\ldots ,U_n$ and their indices are $\\alpha_1 < \\ldots <\\alpha_n$.\r\nLet $p_1,\\ldots ,p_n$ be rational points in $U_1,\\ldots ,U_n$ respectively. Choose rationals $s_i,t_i$ such that $s_1 < \\alpha_1 < t_1 < \\alpha_2 < \\ldots < \\alpha_n < t_n$\r\nConsider $f \\in Q$ the function that $f(s_i,t_i) = p_i$ and $0$ otherwise. It is easy to see that $f \\in \\Pi {\\{{U_{\\alpha}\\}}_{\\alpha \\in \\mathbb{R}}}$ because $f(\\alpha_i) = p _i\\in U_i$ and for other values $\\alpha$ we have $ f(\\alpha) \\in U_{\\alpha}$ because $U_{\\alpha} = \\mathbb{R}$",
"Solution_9": "That looks like it works. This is still a monster.\r\n\r\nThe theorem proved here: if $A$ is separable and $B$ has a Hausdorff topology with a countable base, $A^B$ is separable in the product topology. This probably restricts the cardinality of $B$ to no more than that of the real numbers; I don't believe there are any separable (and Hausdorff) topological spaces larger in cardinality than $\\mathbb{R}^\\mathbb{R}.$",
"Solution_10": "[quote=\"jmerry\"]I don't believe there are any separable (and Hausdorff) topological spaces larger in cardinality than $\\mathbb{R}^\\mathbb{R}.$[/quote]\r\nThat's right and easy to prove. Let $X$ be a Hausdorff space and let $D$ be a countable dense subset of $X$. For each point $x\\in X$ consider the set of all subsets of $D$ of the form $U\\cap D$ where $U$ is a neighborhood of $x$ in the topology of $X$. It is easy to check that the corresponding families of subsets are different for different $x$. Thus the cardinality of $X$ does not exceed that of $P(P(D))$, which is the cardinality of $\\mathbb R^{\\mathbb R}$."
}
{
"Tag": [],
"Problem": "is anyone else taking idea math? which class?\r\nI'm taking \"Math Circles\"",
"Solution_1": "I might take Math circles at session 2."
}
{
"Tag": [],
"Problem": "hi all, pls help mi with this... :) \r\n\r\nwhy does the cis isomer has lower melting point but higher boiling point, solubility and dipole moment than the trans isomer?\r\n\r\n\r\nthx lots",
"Solution_1": "Welcome to this forum.\r\n\r\nThe molecules of a cis isomer, as they are less simetric relative to the molecules of the trans isomer, don't pack so readily in an ordered crystaline structure, and so their melting points are, in general, higher.\r\n\r\nIn the cis isomer, the dipole moments of the two cis bonds add, whereas in trans isomer they cancel each other (this is specially true when the groups are the same). So, cis isomer has a greater dipole moment, i.e., is more polar. This higher polarity os cis isomer molecules explain why they have higher boiling points (stronger intermolecular forces) and better solubilities (stronger interactions with solvent)."
}
{
"Tag": [
"number theory unsolved",
"number theory"
],
"Problem": "find all [b]x,y[/b] and [b]z[/b] in positive integer:\r\n[b]1!+2!+...+x!=y z. [/b]",
"Solution_1": "I think you mean $z \\geq 2$ otherwise there are many solutions :D \r\nAssume that $ z \\geq 2$ and $x >1$ (we clearly have that $(1,1,z)$ is a solution).\r\nLet $S(x)= 1! + \\cdots + x!$\r\n\r\n- If $z = 2$ : Then for $i \\geq 5$, we have $i! = 0 \\mod [10]$. Thus, if $x \\geq 4$, we have $S(x) = S(4) = 3 \\mod [10]$. But no square has unit digit $3$, so there is no solution in that case.\r\nDirect checking leads to the only solution $(3,3,2)$.\r\n\r\n- If $z \\geq 3$ : As above, we see that for $x \\geq 2$ we have $S(x) = 0 \\mod [3]$, so that $3$ has to divide $y$. But, since $ z \\geq 3$, it follows that $S(x) = 0 \\mod [27]$. It is easy to verify that this cannot happen when $x \\geq 8$.\r\nDirect checking shows that there is no solution for $x \\leq 7$, and we are done.\r\n\r\nTherefore, the solutions are $(x,y,z) = (1,1,z)$ and $(3,3,2)$.\r\n\r\nPierre."
}
{
"Tag": [
"geometry",
"inequalities",
"trigonometry",
"inequalities unsolved"
],
"Problem": "Prove that for all acute triangle holds\r\n$\\sqrt3R\\geq\\frac{a^{2}+b^{2}+c^{2}}{a+b+c}.$",
"Solution_1": "[quote=\"arqady\"]Prove that for all acute triangle holds\n$\\sqrt3R\\geq\\frac{a^{2}+b^{2}+c^{2}}{a+b+c}.$[/quote]\r\n\r\nI think it is not true.. Try $A \\rightarrow \\frac{\\pi}{2}$ and $B=C$.",
"Solution_2": "[quote=\"Sung-yoon Kim\"][quote=\"arqady\"]Prove that for all acute triangle holds\n$\\sqrt3R\\geq\\frac{a^{2}+b^{2}+c^{2}}{a+b+c}.$[/quote]\n\nI think it is not true.. Try $A \\rightarrow \\frac{\\pi}{2}$ and $B=C$.[/quote]\r\nIf $a=\\sqrt2$ and $b=c=1$ then $R=\\frac{1}{\\sqrt2}$ and\r\n$\\sqrt3R\\geq\\frac{a^{2}+b^{2}+c^{2}}{a+b+c}\\Leftrightarrow\\sqrt{\\frac{3}{2}}\\geq\\frac{4}{2+\\sqrt2},$ which true.",
"Solution_3": "What about $A \\to 0$, $B=C$? Then $b=c$ and $a \\to 0$, but $R \\to \\frac{b}{2}$, right? (I suck at geometry so I hope I haven't misunderstood notation.) Then your expression approaches $\\frac{\\sqrt{3}}{2}> 1$, which false.",
"Solution_4": "Thank you, Xevarion.",
"Solution_5": "[color=darkblue][b]Remark.[/b] Suppose the isosceles triangle $ABC$ ($b=c$). If $a\\searrow 0$, then $b=c\\nearrow 2R$.\nThus, $\\frac{a^{2}+b^{2}+c^{2}}{a+b+c}=\\frac{a^{2}+2b^{2}}{a+2b}\\rightarrow b\\rightarrow 2R>R\\sqrt 3\\ !$ For example, the triangle $(1,4,4)\\ .$\n\n[b]Proposed problem.[/b] Let $ABC$ be a triangle for which there is $0<2r =2,we can prove when c is large enough,\r\nP(Gc+2^c)>=2^(c+1), :huh: which is a contrariety\r\n\r\nas for deg(P(X))=1 or 0,it's easy :P \r\n that I didn't provethe sentenses end with :huh:. but I think to prove them is not hard",
"Solution_2": ":lol: ,nice approach\r\n 0 isnt natural no.,but i think yours approach may work :D \r\n will tell if get :roll:"
}
{
"Tag": [
"calculus",
"integration",
"trigonometry",
"logarithms",
"LaTeX",
"calculus computations"
],
"Problem": "I know the solution from the back of the book but i cant seem to work it out right.\r\n\r\nInt from pi to 0 : tan[x/2] \r\n\r\nusing the natural log to int. The solution the book gives is ln2. How do I work this out?\r\nThank you!!![/b]",
"Solution_1": "$\\int_0^{\\pi}\\tan\\left(\\frac x2\\right)\\,dx=\\infty.$\r\n\r\nWhat, exactly, is the problem you're looking at?",
"Solution_2": "yeah thats pretty much it only its supposed to be\r\n\r\n$ \\int_0^{\\frac{\\pi}2}\\tan\\left(\\frac x2\\right)\\,dx.$\r\n\r\n[moderator edit]\r\n\r\nfrom 0 to pi/2\r\n\r\nI was working it out using \r\n\r\ntan(u)du \r\nand I wind up with \r\n2ln sec|u| evaluated from 0 to pi \r\n\r\nor as an alternative i get \r\n\r\n-ln cos|u| evauated from 0 to pi \r\n\r\nI am sorry I'm new and I am not familiar with using the displaystyle",
"Solution_3": "Let's just push the substitution through. You're clearly making the substitution\r\n\r\n$u=\\frac x2,\\,x=2u,\\,dx=2du.$\r\n\r\nJust remember now that although $x$ goes from $0$ to $\\frac{\\pi}2,$ $u$ goes from $0$ to $\\frac{\\pi}4.$ That's $\\frac{\\pi}4,$ not $\\pi.$ So we now have\r\n\r\n$2\\int_0^{\\frac{\\pi}4}\\tan u\\,du.$\r\n\r\nFinish it from there.",
"Solution_4": "That helps a lot. Thank You\r\n\r\nI plug in -2ln(cos(pi/4)) - -2ln(cos(0))\r\nand I get .693147... which is equal to ln2\r\n\r\nBut what I dont understand is why the book writes the solution as ln2\r\nis there something that I'm missing. Can that evaluation be simplified to avoid getting the decimal?\r\n\r\nThanks again!!",
"Solution_5": "Tell me: what is $\\cos\\left(\\frac{\\pi}4\\right)$ ? The exact value, not the decimal. This is one of the values you must know.\r\n\r\nSecond issue: is there a way of rewriting $\\ln\\left(\\sqrt{u}\\right)$ so that it doesn't have any square root symbols? And can you simplify $\\ln\\left(\\frac1u\\right)$?",
"Solution_6": "We just started going over natural logs in class so I'm a little shaky with some of the specifics of simpilfying and finding dy/dx and ints, but this is how I approached it.\r\n\r\ncos(pi/4) = :rt2: / 2\r\n\r\nthen: -2ln( :rt2: /2) \r\n= -2ln( :rt2: ) - -ln(2)\r\n= -2ln(2^(1/2)) - -ln2\r\n= -ln2 - -ln2 ?\r\nI must be doing something wrong.\r\nAs for the ln(1/u) cant that be written as\r\nln1-ln(u)\r\nIs that what you meant?",
"Solution_7": "[quote=\"illcrowflu\"]That helps a lot. Thank You\n\nI plug in -2ln(cos(pi/4)) - -2ln(cos(0))\nand I get .693147... which is equal to ln2\n\nBut what I dont understand is why the book writes the solution as ln2\nis there something that I'm missing. Can that evaluation be simplified to avoid getting the decimal?\n\nThanks again!![/quote]Just enjoying this--\r\nThe right hand gives you zero--from the cos $\\rightarrow$ 1; ln 0[b].[/b]\r\nThe left, a multiple of ln(cos(:pi:/4)) --by -2 -- can be rewritten with that [b]multiplier in the exponent[/b] ([b]inside the log[/b]). ~Square 1/ (1/:sqrt:2)... Tada. But I'm not familiar to this forum, so I might well be clueless here.\r\n :downto:\r\n\r\nP.S. Yeah, I hope I wasn't giving anything too much away.",
"Solution_8": "[quote]As for the ln(1/u) cant that be written as\nln1-ln(u) [/quote]\r\nI'd never let a calculus student of mine get away with leaving $\\ln1$ in an answer. Come on, now, $\\ln1=0.$ What I wanted you to say was that $\\ln\\left(\\frac1u\\right)=\\ln(u^{-1})=-\\ln(u).$\r\n\r\n$-2\\ln\\left(\\frac{\\sqrt{2}}2\\right)=-2\\ln\\left(\\frac1{\\sqrt{2}}\\right) = -2\\ln(2^{-1/2}) = -2\\left(-\\frac12\\right)\\ln2=\\ln2.$\r\n\r\nfedwini, I'd edit your bad LaTeX code to make it readable if I could figure out what you were trying to say, but I can't. It looks like you're trying to use LaTeX interchangably with the emoticons just as a collection of symbols. That's not a good way of doing it. Try writing expressions entirely in LaTex. For instance, (dollar)\\frac{\\sqrt{2}}{2}(dollar) becomes $\\frac{\\sqrt{2}}{2}$.",
"Solution_9": "Thanks a lot Kent you were a big help - I appreciate your time.\r\nBTW I am new to the forum but I plan to keep posting as it has been so helpful.\r\n\r\nFor everyone else's sake how do I write the equations out like you were rather than using regular ascii characters?",
"Solution_10": "[quote]For everyone else's sake how do I write the equations out like you were rather than using regular ascii characters?[/quote]\r\nIt's called LaTeX. There's some information posted over on the side, as one of the site guides, and there's also a LaTeX forum for questions and problem.\r\n\r\nGo look at your own #3 post on this topic, which I edited, to get an idea.\r\n\r\nA dollar sign puts you into LaTeX mode, and another dollar sign gets you out. Within that, a backslash introduces a named object, with the names mostly being mnomonic: \\int for integral, \\infty for infinity, \\frac for fraction, and so on. ^ is for superscript, _ for subscript. Curly braces {} delineate and enclose the scope of each object. Thus \\frac{x^2+1}{2x+e^{x-1}} becomes $\\frac{x^2+1}{2x+e^{x-1}}.$",
"Solution_11": "[quote=\"Kent Merryfield\"][quote]$\\ldots$\nfedwini, I'd edit your bad LaTeX code to make it readable if I could figure out what you were trying to say, but I can't. It looks like you're trying to use LaTeX interchangably with the emoticons just as a collection of symbols. That's not a good way of doing it. Try writing expressions entirely in LaTex. For instance, (dollar)\\frac{\\sqrt{2}}{2}(dollar) becomes $\\frac{\\sqrt{2}}{2}$.[/quote][/quote]I've got that fixed now--and I do need to use better LaTeX. There is one thing that I wanted to mention, however: (illcrowflu,) Hovering over any LaTeX picture shows the coding, but ~ If you cannot get that to work for you with long strings, quoting someone else's message will always show you what they wrote--though you'd intend to cancel out of the Reply.",
"Solution_12": "Actually, selecting and copying that part of the post should work. If you paste that in, you'll get the code, without the dollar signs.",
"Solution_13": "[quote=\"jmerry\"]Actually, selecting and copying that part of the post should work. If you paste that in, you'll get the code, without the dollar signs.[/quote]Good deal--I mean good to know. :lol:"
}
{
"Tag": [
"function",
"floor function",
"algebra proposed",
"algebra"
],
"Problem": "Let $ [x]$ denote the integer part of $ x$, i.e., the greatest integer not exceeding $ x$. If $ n$ is a positive integer, express as a simple function of $ n$ the sum \r\n\r\n$ \\left[\\frac{n\\plus{}1}{2}\\right] \\plus{} \\left[\\frac{n\\plus{}2}{4}\\right]\\plus{}...\\plus{}\\left[\\frac{n\\plus{}2^k}{2^{k\\plus{}1}}\\right]\\plus{}...$\r\n\r\n\r\nBig hint (try to solve it w/o):\r\n\r\n[hide]$ \\lfloor{x\\plus{}\\frac{1}{2}}\\rfloor \\equal{} \\lfloor{2x}\\rfloor\\minus{}\\lfloor{x}\\rfloor$[/hide]",
"Solution_1": "http://www.artofproblemsolving.com/Forum/viewtopic.php?t=125439\r\nhttp://www.artofproblemsolving.com/Forum/viewtopic.php?t=34668\r\n\r\nThere's also at least one other thread where this problem was posted in which I gave a slightly different solution. \r\n\r\nI also think it would be very instructive, although overkill, to give a proof by [b]generating functions.[/b]"
}
{
"Tag": [
"algebra unsolved",
"algebra"
],
"Problem": "$ \\begin {cases} x^2\\plus{}y^2\\equal{}z^2\\\\xz\\equal{}y^2\\\\xy\\equal{}10\\end {cases}$",
"Solution_1": "[quote=\"thanhnam2902\"]$ \\begin {cases} x^2 \\plus{} y^2 \\equal{} z^2 \\\\\nxz \\equal{} y^2 \\\\\nxy \\equal{} 10\\end {cases}$[/quote]\r\n\r\nSee that $ x,y,z$ are not equal to zero. Then \r\n\\[ \\Rightarrow x^2 \\plus{} xz \\equal{} z^2 \r\n\\]\r\n\\[ \\Leftrightarrow t^2 \\plus{} t \\minus{} 1 \\equal{} 0,t \\equal{} \\frac{x}{z}\r\n\\]\r\n\\[ \\Rightarrow \\left[ \\begin{array}{l}\r\n t \\equal{} \\frac{{ \\minus{} 1 \\plus{} \\sqrt 5 }}{2} \\\\ \r\n t \\equal{} \\frac{{ \\minus{} 1 \\minus{} \\sqrt 5 }}{2} \\\\ \r\n \\end{array} \\right.\r\n\\]",
"Solution_2": "[quote=\"Honey_S\"][quote=\"thanhnam2902\"]$ \\begin {cases} x^2 \\plus{} y^2 \\equal{} z^2 \\\\\nxz \\equal{} y^2 \\\\\nxy \\equal{} 10\\end {cases}$[/quote]\n\nSee that $ x,y,z$ are not equal to zero. Then\n\\[ \\Rightarrow x^2 \\plus{} xz \\equal{} z^2\n\\]\n\n\\[ \\Leftrightarrow t^2 \\plus{} t \\minus{} 1 \\equal{} 0,t \\equal{} \\frac {x}{z}\n\\]\n\n\\[ \\Rightarrow \\left[ \\begin{array}{l} t \\equal{} \\frac {{ \\minus{} 1 \\plus{} \\sqrt 5 }}{2} \\\\\nt \\equal{} \\frac {{ \\minus{} 1 \\minus{} \\sqrt 5 }}{2} \\\\\n\\end{array} \\right.\n\\]\n[/quote]\r\nThank you. Oh! This system is easy."
}
{
"Tag": [
"probability",
"function",
"expected value",
"probability and stats"
],
"Problem": "Brief interruptions to my work occur on average every ten minutes and the number of interruptions in any given time period has a Poisson distribution. Given that an interruption has just occurred, find the probability that I will have less than t minutes to work before the next interruption. If the random variable T is the time I have to work before the next interruption, find the probability density function of T.\r\n\r\nI need an uninterrupted half hour to finish an important paper . Show that the expected number of interruption before my first uninterrupted period of half an hour or more is $ e^3 \\minus{}1$. Find also the expected length of time between interruptions that are less than half an hour apart. Hence write down the expected wait before my first uninterrupted period of half an hour or more.",
"Solution_1": "if you have already heard about Poisson processes, you immediately know that the time length between two consecutive interruptions is exponentially distributed with mean $ 10$ minutes. From here, everything is straightforward.\r\n1: $ T$ has density $ \\lambda \\exp(\\minus{}\\lambda t) \\, dt$ on $ (0,\\infty)$ with $ \\lambda \\equal{} \\frac{1}{10 \\text{ min}}$\r\n2: properties of the geometric distribution gives you the expected number of interruption before the first uninterrupted period of half an hour or more, etc ...\r\n\r\nIs this OK ?",
"Solution_2": "I'm not familiar with the geometric distribution, but I solved it like this:\r\n\r\n$ p \\equal{} P(T > 30) \\equal{} e^{ \\minus{} 3}$\r\n\r\nExpected number of interruptions before the first uninterrupted half hour:\r\n\r\n$ 0\\cdot p \\plus{} 1\\cdot (1 \\minus{} p) p \\plus{} 2 \\cdot (1 \\minus{} p)^2 \\cdot p \\plus{} 3 \\cdot (1 \\minus{} p)^3 \\cdot p \\plus{} \\cdots \\equal{} p (\\frac {1}{p^2} \\minus{} \\frac {1}{p}) \\equal{} e^3 \\minus{} 1$\r\n\r\nFor the expected time to pass before the first uninterrupted half hour I calculated about 2h 40min. What's your solution?\r\nAre you studying in Cambridge?",
"Solution_3": "1: yes, your solution is essentially what I had in mind (http://en.wikipedia.org/wiki/Geometric_distribution)\r\n2: For the expected time to pass before the first uninterrupted half hour it is the same idea:\r\n\\[ \\equal{} E[T|T < 30] \\cdot p\\plus{}2\\cdot E[T|T < 30] \\cdot (1\\minus{}p) p\\plus{}3 \\cdot E[T|T < 30] \\cdot (1\\minus{}p)^{2}\\cdot p\\plus{}... \\]"
}
{
"Tag": [],
"Problem": "I'm not sure if this is the right forum.Anyway here's the question:-\r\nIn a convex quadrileteral ABCD the angle bisectors of angle ABC and angle CAB meet at point [b]P[/b].The angle bisectors of angle BAD and angle DBA meet at [b]Q[/b].Prove that if A,B,C,D are concyclic then so is A,B,P,Q.",
"Solution_1": "Since $ ABCD$ is cyclic, $ \\angle DAC \\equal{} \\angle DBC \\equal{} x$. Then, letting $ \\angle CAB \\equal{} y$, $ \\angle DBA \\equal{} z$, we see through simple angle chasing that \r\n\r\n$ \\angle BPA \\equal{} 180 \\minus{} \\frac{\\alpha\\plus{}\\beta\\plus{}\\gamma}{2} \\equal{} \\angle BQA$. So $ ABPQ$ is cyclic."
}
{
"Tag": [
"function",
"algebra unsolved",
"algebra"
],
"Problem": "Find all functions $f(n)$ such that $f(2n) = f(n+1)^2 - f(n-1)^2$\r\n\r\n1) $f : \\mathbb{N} \\rightarrow \\mathbb{N}$\r\n2) $f : \\mathbb{R} \\rightarrow \\mathbb{R}$",
"Solution_1": "Is it really an olympiad problem?? :maybe: \r\n\r\nI'm getting a lot of weird stuff, yet I still can't manage to conclude.\r\n\r\nFor the 1st one, I think there are none, but I can't prove it. (I can tell you what I have so far)\r\n\r\nFor the 2nd one, I guess there are infinitely many, but I don't think there's an easy characterization. (I can show you a few examples)"
}
{
"Tag": [
"MATHCOUNTS"
],
"Problem": "Lets talk about this game of extremes.\r\nEither you know about it, or dont, and love it, or hate it.\r\nPost igns, character info, and try to convince me to quit.\r\n ;)",
"Solution_1": "LOL, you could do homework with all that time. :)",
"Solution_2": "well, i only play on weekends, so if you're implying me doing extra work upon extra work... :P \r\n\r\nany other arguements?",
"Solution_3": "what is maplestory? :?",
"Solution_4": "Its an addicting free online mmorpg much like runescape, though i've never played runescape. Its 2D sidescroll, and may sound shabby at first, but most definitely isn't. I would emphasize the addicting part.",
"Solution_5": "So I'm guessing that your avatar is your Maplestory character?",
"Solution_6": "not my character. just a random avatar i found. i find that ms character make good avatars... :D",
"Solution_7": "can you give me a link to maplestory?",
"Solution_8": "[quote=\"Stratigizer\"]Lets talk about this game of extremes.\nEither you know about it, or dont, and love it, or hate it.\nPost igns, character info, and try to convince me to quit.\n ;)[/quote]\r\n\r\nOh this game... I tried to start it but it wont get on my computer... I think that its pretty good... a little like runescape... I did make an account though i think. mabey on someone else's com.\r\n\r\nits on http://www.maplestory.com",
"Solution_9": "actually, that site is just a portal.\r\n\r\ngoto http://www.mapleglobal.com for the site, download, and a bit of info.\r\n\r\nits completely free, which adds to the awesomeness.",
"Solution_10": "is there any way to play without downloading it? it would take nearly 5 hours on just the .exe file alone on my computer.",
"Solution_11": "[quote=\"MathFiend\"]is there any way to play without downloading it? it would take nearly 5 hours on just the .exe file alone on my computer.[/quote]\r\n\r\nyea. it says 56 hours for my com.",
"Solution_12": "[quote=\"Spencer\"][quote=\"MathFiend\"]is there any way to play without downloading it? it would take nearly 5 hours on just the .exe file alone on my computer.[/quote]\n\nyea. it says 56 hours for my com.[/quote]\r\n\r\nIm trying to download right now...\r\n\r\nHey, if I download the thing on the bottom, then the main game loads for only likke 1 hour.",
"Solution_13": "I used to play this...\r\nGot to be pretty boring though\r\nI mean killing the same things over and over....\r\nWhat the point??\r\nMy opinion\r\n[hide]Quit before it takes over ur life!!![/hide]\nI got some people at my school who talk about it for hours\nIt like the only thing they do \nThis is better than Maplestory\n[hide] :ddr: [/hide]",
"Solution_14": "If trying to download the game,\r\nDo the parts\r\nIt is way faster\r\nIt takes 4 hours on my comp to download all of it\r\nbut only 2 hours with parts",
"Solution_15": "Well, you only have pretty much 1 choice for armor.\r\nuse the two-handed swords, scimitar and lionheart.\r\ntho they are more expensive, they look cooler and are the most balanced.\r\n\r\nanyways, i always liked magic claw better, as the terrain is no obstacle for attacking. and regular [i]claws[/i] are for theives, yes, the throwing star casters...\r\nanyways, im still 46, doing all these awesome new quests!\r\n[/i]",
"Solution_16": "Well, you only have pretty much 1 choice for armor.\r\nuse the two-handed swords, scimitar and lionheart.\r\ntho they are more expensive, they look cooler and are the most balanced.\r\n\r\nanyways, i always liked magic claw better, as the terrain is no obstacle for attacking. and regular [i]claws[/i] are for theives, yes, the throwing star casters...\r\nanyways, im still 46, doing all these awesome new quests!",
"Solution_17": "Well, you only have pretty much 1 choice for armor.\r\nuse the two-handed swords, scimitar and lionheart.\r\ntho they are more expensive, they look cooler and are the most balanced.\r\n\r\nanyways, i always liked magic claw better, as the terrain is no obstacle for attacking. and regular [i]claws[/i] are for theives, yes, the throwing star casters...\r\nanyways, im still 46, doing all these awesome new quests!",
"Solution_18": "Well, you only have pretty much 1 choice for armor.\r\nuse the two-handed swords, scimitar and lionheart.\r\ntho they are more expensive, they look cooler and are the most balanced.\r\n\r\nanyways, i always liked magic claw better, as the terrain is no obstacle for attacking. and regular [i]claws[/i] are for theives, yes, the throwing star casters...\r\nanyways, im still 46, doing all these awesome new quests!",
"Solution_19": "umm... spam???\r\n\r\nwhat is wrong with your posts?",
"Solution_20": "WOAA!! I've seen plenty of double posts, but never a quadruple post :oops_sign: :w00tb: \r\n\r\n\r\nHehe I'm a hardcore RS fan.\r\n\r\nMaplestory has only one skill: BEATING CUTE FUZZY THINGS TILL THEY YELL :help: \r\n\r\nSeems a bit too simple for me. On the other hand RS has 21 skills.\r\n\r\nFarmers for life.",
"Solution_21": "LVL 57 finally ice-lightning if u guys forgot\r\nI got 3 anakamoon\r\nIf anyone is on scania, I may give u guys one.\r\nmy name is [i]yomama789[/i]\r\nDo any of u guys have BC ores, I need 50 of them and I onli get 20 right now",
"Solution_22": "[quote=\"Stratigizer\"]Well, you only have pretty much 1 choice for armor.\nuse the two-handed swords, scimitar and lionheart.\ntho they are more expensive, they look cooler and are the most balanced.\n[/quote]\r\n\r\nI'm 31, blunt weapon page.\r\nAnd you say I have only 1 choice, but I'm still not too knowing of everything MapleStory, could you tell me which armor I should get? Lvl requirement 30 or 35, stuff I'd keep until 40.\r\nAnd I have lvl 25 armor, if there's any you think would be worth keeping until 40, please advise!",
"Solution_23": "um, yeah, i have no clue what happened there...sorry about that!\r\n\r\nim not sure about the stat requirements, but at lv 30, your top is the green hwarang shirt or w/e you call them, black martial arts pants, the shoe you get from nella's last quest, and it doesn't really matter the glove.\r\n\r\nlv 35 you should get brown/blue helm, forgot the name, and white/black chainmail, which is overall. keep your lv 30 shoes, and you should have lv 35 gloves from john's sleepywood jump quest.\r\n\r\nIf you have an orihalcon burgernet helm, keep it til 35, and if you have venon, or perhaps the lv 25 glove, keep until 35. however, i dont think you should keep anything til lv 40. stat-wise, its not beneficial compared to cost, and it just doesn't look as cool!\r\n\r\ngoto http://www.hidden-street.net to look at the stats and requirements, but also, to everyone who doesn't know, goto http://www.happymapling.net for a look at equips up to lv 100!\r\n\r\nok, enough of that...",
"Solution_24": "Ludi's coming out soon\r\nI'm so prepared for it\r\nif no one believes me go to mapleglobal to look",
"Solution_25": "so...what do you think of ludi?\r\n\r\n48 now",
"Solution_26": "It's okay, I want to try the pq though.\r\nI uninstalled MapleStory because it was distracting me from mathcounts -_-",
"Solution_27": "pq is what, 35-50?\r\nbut it seems like the other pq, it just takes too long to get in ><\r\n\r\nI guess state hasn't happened yet? Anyways, good luck! and I'm from Texas too, so go Texas ;)",
"Solution_28": "i love maple lol\r\n\r\nthe best part of maple is calculating profits regarding scrolling and junk, i'm soo rich from just doing nothing but scrolling. \r\n15dex 6 slots left sauna robe \r\n11 att gloves \r\netc XD",
"Solution_29": "yeah, i too just discovered my lucky hand in scrolling. my overall robe is +6, with 3 10% def scrolls. i also have +12 jump +4speed shoes, with like 2 10% jump scrolls...\r\nalso, my friend borrowed a fish spear and tried 3 10% for spear att, and they all worked.\r\n\r\nyeah, seems like its gonna be all 10% for me =)"
}
{
"Tag": [
"algebra",
"polynomial",
"superior algebra",
"superior algebra unsolved"
],
"Problem": "Prove that if [b]K[/b] is a field and [i]x_1, x_2, ... x_n[/i] are elements of algebric closer of [b]K[/b], then [b]K[/b]([i]x_1, x_2, ... x_n[/i]) is a collection of all the expressions of the form [b]f[/b]([i]x_1, x_2, ... x_n[/i]) in wich [b]f[/b] beloning to [b]K[X_1,...X_n][/b] is a polynomial such that [i]deg[/i](X_i) [b]f[/b] < [b][K([i]x_i[/i]):K][/b]",
"Solution_1": "Induction."
}
{
"Tag": [],
"Problem": "The rules of this game are as follows:\r\n\r\n1. Each player gets to pick 5 things they would have with them if they were stranded on a barren wasteland. These items cannot be larger than a house.(e.g. a tent would be a good idea)\r\n2. players can trade items by typing the username, the item that they want, and the item they are willing to trade from the person they want to trade with. The person can then decide whether to trade or not.\r\n3. every week or so I will post a problem. Each player will PM me their solutions to me, and the players who get the problem wrong must give up an item.\r\n4. The players with insufficient items for survival will be eliminated.(I will be the one to decide who has insufficient items)\r\n5. There will be a max of 5 players per game.\r\n6. The winner of this game may host the next one.\r\n\r\n[hide=sign ups][/hide]",
"Solution_1": "Sign up...",
"Solution_2": "sign up\r\n\r\n[hide=\"signups\"]\nBugi\nwestiepaw\n[/hide]",
"Solution_3": "sign up\r\n\r\n[hide=\"signups\"]\nBugi\nwestiepaw\nspaceguy524\n[/hide]",
"Solution_4": "Sign up.\r\n\r\n[hide=\"signups\"]\nBugi\nwestiepaw\nspaceguy524\ncyberspace\n[/hide]\r\n\r\nOne more...",
"Solution_5": "[b]sign up[/b]",
"Solution_6": "WAIT! Never mind. There's already 5 people. :P",
"Solution_7": "o.k.... The competition begins.\r\n\r\nYou 5... Pick your 5 items, and be smart about them. If you're not smart about them, you might just die when the game starts. :)",
"Solution_8": "What does the barren island have on it? Is there any hope of escape, or should I choose items for long-term survival, rather than escape? Thanks!",
"Solution_9": "Drop out, sorry.",
"Solution_10": "Ok, i will sign up since Bugi dropped out."
}
{
"Tag": [
"geometry",
"trapezoid"
],
"Problem": "Let a circle be inscribed in an isosceles trapezoid with base lengths $ m$ and $ n$. Express the diameter $ d$ of the circle in terms of $ m$ and $ n$.",
"Solution_1": "[quote=\"123456789\"]Let a circle be inscribed in an isosceles trapezoid with base lengths $ m$ and $ n$. Express the diameter $ d$ of the circle in terms of $ m$ and $ n$.[/quote]\r\n\r\n[hide]\nDraw the segments from the center of the circle to each vertex of the trapezoid, and the perpendiculars to each edge.\nIt's easy to see that the lateral sides must be $ \\frac {m \\plus{} n}{2}$ by congruencies.\nDraw the height from the upper left vertex. Use the pythagorean theorem.\n$ \\frac {m \\minus{} n}{2}^2 \\plus{} h^2 \\equal{} \\frac {m \\plus{} n}{2}^2$\n$ \\frac {m^2 \\plus{} n^2}{4} \\minus{} \\frac {mn}{2} \\plus{} h^2 \\equal{} \\frac {m^2 \\plus{} n^2}{4} \\plus{} \\frac {mn}{2}$\ncancel, combine like terms\n$ h^2 \\equal{} mn$\nor in other words\n$ (2r)^2 \\equal{} mn$\n$ r \\equal{} \\frac {\\sqrt {mn}}{2}$\n[/hide]\r\n\r\ntoo simple..someone tell me what i did wrong\r\n\r\nedit: yes alan i meant circle",
"Solution_2": "uh, triangle???",
"Solution_3": "[quote=\"alanchou\"]uh, triangle???[/quote]\r\n\r\noh heh that was a typo, a freudian slip to be exact :rotfl:",
"Solution_4": "[quote=\"CircleSquared\"][quote=\"123456789\"]Let a circle be inscribed in an isosceles trapezoid with base lengths $ m$ and $ n$. Express the diameter $ d$ of the circle in terms of $ m$ and $ n$.[/quote]\n\n[hide]\nDraw the segments from the center of the circle to each vertex of the trapezoid, and the perpendiculars to each edge.\nIt's easy to see that the lateral sides must be $ \\frac {m \\plus{} n}{2}$ by congruencies.\nDraw the height from the upper left vertex. Use the pythagorean theorem.\n$ \\frac {m \\minus{} n}{2}^2 \\plus{} h^2 \\equal{} \\frac {m \\plus{} n}{2}^2$\n$ \\frac {m^2 \\plus{} n^2}{4} \\minus{} \\frac {mn}{2} \\plus{} h^2 \\equal{} \\frac {m^2 \\plus{} n^2}{4} \\plus{} \\frac {mn}{2}$\ncancel, combine like terms\n$ h^2 \\equal{} mn$\nor in other words\n$ (2r)^2 \\equal{} mn$\n$ r \\equal{} \\frac {\\sqrt {mn}}{2}$\n[/hide]\n\ntoo simple..someone tell me what i did wrong\n\nedit: yes alan i meant circle[/quote]It's meant to be simple. It's High School Basics. And I was asking for $ d$, but $ r$ is just as good."
}
{
"Tag": [
"inequalities",
"inequalities proposed"
],
"Problem": "$ 1)$ Given $ a, b, c \\geq\\ 0$ and $ a\\plus{}b\\plus{}c\\equal{}3$. \r\nProve that: $ \\frac {7a^2 \\plus{} 9}{b^2 \\plus{} c^2} \\plus{} \\frac {7b^2 \\plus{} 9}{c^2 \\plus{} a^2} \\plus{} \\frac {7c^2 \\plus{} 9}{a^2 \\plus{} b^2}\\ge 24$\r\n$ 1)$ Given $ a, b, c \\geq\\ 0$ and $ ab\\plus{}bc\\plus{}ca\\equal{}3$. \r\nProve that: $ \\frac {4a^2 \\plus{} 3}{b^2 \\plus{} c^2} \\plus{} \\frac {4b^2 \\plus{} 3}{c^2 \\plus{} a^2} \\plus{} \\frac {4c^2 \\plus{} 3}{a^2 \\plus{} b^2}\\ge \\frac {21}{2}$\r\n\r\nI see some long proofs in here: http://diendantoanhoc.net/forum/index.php?showtopic=45718\r\nIndeed, both of them can be solve easily by CS and Schur ineq :)",
"Solution_1": "I'm also a fan of Vasc's problems , note that the first and the second one are equivalents so I only write the solution of the frst one:\r\n\r\nit's equivalent to:\r\n\r\n$ 7\\sum{ \\frac{a^2}{b^2+c^2}} +( \\sum{a})^2 \\sum{\\frac{1}{b^2+c^2}} \\geq 24$\r\n\r\nusing Cauchy it follows that :\r\n\r\n$ 7\\sum{ \\frac{a^2}{b^2+c^2}} \\geq 7\\frac{(\\sum{a^2})^2}{2\\sum{(ab)^2}}$\r\n\r\nand $ \\sum{ \\frac{1}{b^2+c^2} } \\geq \\frac{ 3+ \\frac{(\\sum{a^2})^2}{2\\sum{(ab)^2}}{\\sum{a^2}}$\r\n\r\nWLOG $ a+b+c=1$ it's equivalent to :\r\n$ f(r)= 4-23q+23q^2+20q^3+42r-96rq \\geq 0$ and thus f is increasing\r\nand it's easy to check that : if $ q\\leq \\frac{1}{4}$ $ f(0 )\\geq 0$ and if $ q\\geq \\frac{1}{4}$ $ f(\\frac{4q-1}{9}) \\geq 0$"
}
{
"Tag": [],
"Problem": "Are given 25 numbers . The sum of each 4 of them is positive prove that the sum of all those numbers is positive .",
"Solution_1": "can't we just add all the \"4-possibilities\" up? :maybe: \r\nbecause then each number would appear $ \\binom{24}{3}$ times,I mean let $ x_1,x_2,\\ldots ,x_{25}$ be our 25 numbers,and we had:\r\n\r\n$ x_1 \\plus{} x_2 \\plus{} x_3 \\plus{} x_4 \\equal{} a_1$\r\n.\r\n.\r\n.\r\n$ x_{22} \\plus{} x_{23} \\plus{} x_{24} \\plus{} x_{25} \\equal{} a_{\\binom{25}{4}}$\r\n\r\nwhere $ a_i$ are positive numbers for $ i \\equal{} 1,2,\\ldots ,\\binom{25}{4}$\r\n\r\nnow add all the equalities we had above,we would end up with:\r\n\r\n$ \\binom{25}{3}\\sum_{i \\equal{} 1}^{25} x_i \\equal{} \\sum_{i \\equal{} 1}^{\\binom{25}{4}} a_i > 0$\r\n\r\n$ \\Rightarrow \\sum_{i \\equal{} 1}^{25} x_i > 0$\r\n :maybe:"
}
{
"Tag": [
"inequalities",
"geometry",
"perimeter",
"rectangle",
"triangle inequality",
"Pythagorean Theorem"
],
"Problem": "A convex polygon does not extend outside a square side 1. Prove that the sum of the squares of its sides is at most 4.",
"Solution_1": "[hide=\"Hint\"]\nDivide into right triangles. Then use the fact that $ x^{2}\\leq x \\ \\forall x \\leq 1$.[/hide]",
"Solution_2": "[hide=\"Interesting\"]Consider the orthagonal components of each side respective to the square. By the triangle inequality, $ a+b \\geq \\sqrt{a^{2}+b^{2}}$ for component lengths $ a$ and $ b$. Note that the sum of the lengths of these components equals the perimeter of a binding rectangle of that polygon.\n\nLet us orient our components such that the achieved binding rectangle has edges parallel to those of the square. This rectangle must be fully contained within the square, and hence no component can have length greater than 1. It follows that the square of the length of a component cannot exceed the length of that component.\n\nNow, note that the sum of the squares of the side lengths equals the sum of the squares of the lengths of the components (Pythagorean Theorem), which does not exceed the sum of the lengths of the components, which is equivalent to the perimeter of the binding rectangle.\n\nSince neither the length nor width of this binding rectangle can exceed 1, the perimeter of this rectangle cannot exceed 4. Transitively, the sum of the squares of the side-lengths of the polygon cannot exceed 4.\n\nTesting the unit square case gives that this bound is indeed achievable, so 4 is the best upper bound. [/hide]\r\n\r\n...\r\nBah my computer stalled. Anyway, it turns out that this is basically what cinco suggested."
}
{
"Tag": [
"function",
"algebra unsolved",
"algebra"
],
"Problem": "f(x+y)=f(x)f(y) and f(a)=1990. Prove that f is injectiv.",
"Solution_1": "I can't see why you can't take $ f(x) \\equal{} e^{g(x)}$, where $ g(x)$ is a pathological solution of the Cauchy equation with some of the Hamel basis elements (that don't intersect with $ a$) set to $ 0$ by $ g$.",
"Solution_2": "You can set that only if this funciton is monotone or injectiv.",
"Solution_3": "Why? The function $ f: \\mathbb R\\to\\mathbb R$ can be shown to have values in $ \\mathbb R^\\plus{}$ only such that MellowMelon's reasoning does work. Can you provide a proof?"
}
{
"Tag": [
"geometry",
"3D geometry",
"combinatorics unsolved",
"combinatorics"
],
"Problem": "A cube is divided into n^3 little cubes, prove that we can assign a DIFFERENT integer to each little cube so that the sum of each of the 3n^2 rows parallel to an edge is zero.",
"Solution_1": "edit: it means the sum of numbers in the n little cubes in all 3n^2 rows is zero",
"Solution_2": "for $ n\\equal{}2$ it's obviously umtrue :wink: since $ a_{1,1,1} \\plus{} a_{1,1,2} \\equal{} 0$ and also $ a_{1,1,1} \\plus{} a_{1,2,1} \\equal{}0$ so it must be\r\n$ a_{1,2,1} \\equal{} a_{1,1,2}$ which is a contraddiction since the numbers were supposed to be different.",
"Solution_3": "Sorry, the problem states that n >=3"
}
{
"Tag": [
"USAMTS",
"LaTeX",
"modular arithmetic"
],
"Problem": "In general does $ \\sum a\\plus{}b$ mean $ (\\sum a)\\plus{}b$ or $ \\sum (a\\plus{}b)$?",
"Solution_1": "Umm well is this a question that's attempting to get an answer for one of the USAMTS problems?\r\n\r\nIf so, you just have to look it up on your own.",
"Solution_2": "In context, this depends on whether $ b$ has a term that contains the index of the summation. This notation is ambiguous, though; to be precise, you should add parentheses appropriately.",
"Solution_3": "Typically, $ \\sum a \\plus{} b \\equal{} (\\sum a) \\plus{} b$. Heh, I see what problem you're looking at... I don't think this counts as cheating though. :)",
"Solution_4": "[quote=\"Brut3Forc3\"]Typically, $ \\sum a + b = (\\sum a) + b$. Heh, I see what problem you're looking at... I don't think this counts as cheating though. :)[/quote]\r\nIf it's not considered cheating for telling this, I believe it's $ \\sum{(a + b)}$ cuz in $ \\text{\\LaTeX}$, when you just type a+b in the brackets for sum, your exact original reading shows up.",
"Solution_5": "Brackets or parenthesis? Brackets usually disappear on everything...",
"Solution_6": "I meant these brackets: { }",
"Solution_7": "[quote=\"gauss1181\"]I meant these brackets: { }[/quote]\r\n\r\nYeah, if you type a latex expression and use brackets as parenthesis, they disappear. The summation notation in latex does not require brackets...\r\n\r\nusually I type \\displaystyle\\sum_{i=1}^5 x^2 rather than usually I type \\displaystyle\\sum_{i=1}^5{x^2}\r\n\r\nand it does the exact same thing.",
"Solution_8": "[quote=\"pythag011\"][quote=\"gauss1181\"]I meant these brackets: { }[/quote]\n\nYeah, if you type a latex expression and use brackets as parenthesis, they disappear. The summation notation in latex does not require brackets...\n\nusually I type \\displaystyle\\sum_{i=1}^5 x^2 rather than usually I type \\displaystyle\\sum_{i=1}^5{x^2}\n\nand it does the exact same thing.[/quote]\r\nHowever, when one of the \\mod things enclose something in brackets, it's also in parenthesis. I think it's \\pmod. 4\\pmod{3 3k4} shows up like this: $ 4\\pmod{3 3k4}$",
"Solution_9": "[quote=\"ProtestanT\"][quote=\"pythag011\"][quote=\"gauss1181\"]I meant these brackets: { }[/quote]\n\nYeah, if you type a latex expression and use brackets as parenthesis, they disappear. The summation notation in latex does not require brackets...\n\nusually I type \\displaystyle\\sum_{i=1}^5 x^2 rather than usually I type \\displaystyle\\sum_{i=1}^5{x^2}\n\nand it does the exact same thing.[/quote]\nHowever, when one of the \\mod things enclose something in brackets, it's also in parenthesis. I think it's \\pmod. 4\\pmod{3 3k4} shows up like this: $ 4\\pmod{3 3k4}$[/quote]\r\n\r\n\\pmod is not a symbol. \\sum is.",
"Solution_10": "Just assume one and solve the problem with it.",
"Solution_11": "[quote=\"t0rajir0u\"]In context, this depends on whether $ b$ has a term that contains the index of the summation. This notation is ambiguous, though; to be precise, you should add parentheses appropriately.[/quote]\r\n\r\nI totally agree. Just use your common sense and knowledge of summation notation. :wink:"
}
{
"Tag": [
"function",
"calculus",
"calculus computations"
],
"Problem": "If I have a contour diagram of:\r\n\r\n[url=http://imageshack.us][img]http://img118.imageshack.us/img118/9608/graphdu7.jpg[/img][/url]\r\n\r\n\r\nthen what would the graph of:\r\n\r\nf(-x,y) = f(x,y)\r\n\r\nand \r\n\r\nf(-x,y) = -f(x,y)\r\n\r\nlook like??\r\n\r\nmy guess is that the first one is the same as the original one translated with the y-axis and the second one translated with the x-axis. Is this right?",
"Solution_1": "[quote=\"EquinoX\"]f(-x,y) = f(x,y) \n...\ntranslated with the y-axis[/quote] Hmm?",
"Solution_2": "[quote=\"JBL\"][quote=\"EquinoX\"]f(-x,y) = f(x,y) \n...\ntranslated with the y-axis[/quote] Hmm?[/quote]\r\n\r\nWhy a Hmmm??",
"Solution_3": "Because neither of the lines I quoted make sense ;)",
"Solution_4": "What I meant translated with the y-axis is pretend that the y-axis is a mirror, so therefore everything to the right of the y-axis will be reflected to the left of the y-axis. That's what I assume \r\n f(-x,y) = f(x,y) looks like and I just want to confirm whether this is correct or not?",
"Solution_5": "Your notation is wrong. You are talking about f(-x,y)=0, not f(-x, y)=f(x,y).. There is a big difference.. Suppose you have f(x,y) = x+y. Then f(-x,y)=-x+y. So, if you plot x+y=0 you get y = -x, and plotting -x+y=0 you get y=x. This goes by your assumption. But f(x,y) = f(-x,y) is x+y = -x+y => x=0, and this has nothing to do with what you are talking about.",
"Solution_6": "Yes what I am trying to say is to find what the function f(-x,y) looks like if f(x,y) is on the picture above..."
}
{
"Tag": [
"ratio",
"geometry",
"geometric transformation",
"rotation",
"power of a point",
"radical axis",
"geometry proposed"
],
"Problem": "Given the triangle $ABC$ we consider the points $X,Y,Z$ such that the triangles $ABZ,BCX,CAZ$ are equilateral, and they don't have intersection with $ABC$. Let $B'$ be the midpoint of $BC$, $N'$ the midpoint of $CY$, and $M,N$ the midpoints of $AZ,CX$, respectively. Prove that $B'N' \\bot MN$.",
"Solution_1": "Cute one :).\r\n\r\nConsider the transformation we get by composing the spiral similarity cetnered at $B$ of angle $\\frac\\pi 6$ and ratio $\\frac{\\sqrt 3}2$ with the rotation of angle $\\frac \\pi 3$ centered at $C$ (the rotation comes first, then the spiral similarity). This transformation is again a spiral similarity of angle $\\frac \\pi 2$. The image of $Y$ is $M$, and the image of $B$ is $N$, which means that the image of the line $BY$ is $MN$, which shows that $BY\\perp MN$. Now, since $BY\\|B'N'$, we get the conclusion.\r\n\r\nP.S. the above even gives the ratio of the lengths of the two segments.",
"Solution_2": "wow! your solutions is Great! Another nice idea is to use the fact that $MN$ is parallel to the line between the centers of $ABZ,BCX$. Hence by radical axis, and since the Fermat point of $ABC$ belongs to $BY$ and to the intersection of the circles $ABZ,BCX$, we get $MN\\bot BY$ and the conclusion follows.",
"Solution_3": "jojo, excelent, es una muy interesante soluci\u00f3n la de grobber.\r\n\r\nUna m\u00ednima generalizaci\u00f3n del problema es:\r\n\r\nGiven the triangle ABC we consider the points A', B', C' such that the triangles BCA', CAB', ABC' are equilateral an externals to ABC. Let Ca, Ba, Ab, Cb, Bc y Ac be the midpoints of BA', A'C, CB', B'A, AC' y C'B. The intersection of BaBc and CaCb is X, the intersection of AbAc and CaCb is Y, and the intersection of BaBc and AbAc is Z. Prove that XYZ is homotetic with the triangle NaNbNc (centers of BCA', CAB', ABC'), with center F and reason 1/2. (perd\u00f3n por mi ingl\u00e9s, is terrible).",
"Solution_4": "pretty easy I think, the rotation (CCA :pi:/3 2) of BY from C is AX, and the rotation (CA :pi:/6 2/ :sqrt: 3) of MN from B is AX, too. So we get it.",
"Solution_5": "3500th post :thumbup: :coolspeak: :clap2: \n\nComplex number bash ftw.\n\n[hide=\"Solution\"]\n[hide=\"Hints from AoPS\"]\n127) Let $A$ be the origin. What must be true about $(m-n)/(n'-b')$ if and only if $\\overline{MN} \\perp \\overline{B'N'}$?\n85) If you're stuck with a ratio that you'd like to show is inamginary, here's a hint. If $\\frac{z_1}{z_2}=\\frac{z_3}{z_4}=k$, then what does $\\frac{z_1b+z_3c}{z_2b+z_4c}$ equal? [/hide]\nLet $A$ be the origin\n\nWe desire: $\\frac{m-n}{b'-n'}$ being imaginary.\n\nWe know: $b-a=\\zeta(z-a)\\implies b=\\zeta*z$\n\nAlso, $x-c=\\zeta(b-c)$ and $y=\\zeta(c)$. \n\nNow we have a HIGHLY computationally intense problem so let's try it. \n\n$m=\\frac{z+a}{2}=\\frac{\\frac{b}{\\zeta}+a}{2}$. Next, $n=\\frac{x+c}{2}=\\frac{2c+\\zeta(b-c)}{2}$. Also, $b'=\\frac{b+c}{2}$ and $n'=\\frac{c+y}{2}=\\frac{c+\\zeta(c)}{2}$.\n\n$\\frac{m-n}{b'-n'}=\\frac{\\frac{b}{\\zeta}-2c-\\zeta(b-c)}{b-\\zeta*c}$ which we need to be imaginary.\n\nThis is the same thing as $\\frac{\\frac{b}{\\zeta}-2c-\\zeta(b)+\\zeta*c}{b-\\zeta*c}$\n\nThis is also the same as $\\frac{b-\\zeta^2(b)-2\\zeta*c+\\zeta^2*c}{b\\zeta-\\zeta^2*c}$\n\nTherefore, we get $\\frac{b(1-\\zeta^2)+c(\\zeta^2-2\\zeta)}{b\\zeta+c(-\\zeta^2)}$\n\nWe now prove $\\frac{1-\\zeta^2}{\\zeta}=\\frac{\\zeta^2-2\\zeta}{-\\zeta^2}$.\n\nThis is crucial in the next part of the problem.\n\n$\\frac{1-\\zeta^2}{\\zeta}=\\frac{\\zeta-2}{-\\zeta}$ or $1-\\zeta^2=-\\zeta+2$ Therefore, we get $\\zeta^2-\\zeta+1=0$ which is true by the definition of $\\zeta$.\n\nYay, lemma proven.\n\nFind $\\frac{bz_1+cz_3}{bz_2+cz_4}$ where $\\frac{z_1}{z_2}=\\frac{z_3}{z_4}$.\n\nWe note that letting this value be $k$, we get $z_1=kz_2$.\n\nTherefore, we have $\\frac{bkz_2+ckz_4}{bz_2+cz_4}=k$.\n\nTherefore, we must now prove that $\\frac{1-\\zeta^2}{\\zeta}$ is imaginary.\n\nSince $z$ is a primitive 6th root of unity, we get $z=e^{\\frac{\\pi}{3}i}$ or $z=e^{\\frac{5\\pi}{3}i}$. \n\nTherefore, $z=\\frac{\\sqrt3}{2}+\\frac{i}{2}$ or $z=\\frac{1}{2}-\\frac{i\\sqrt3}{2}$.\n\nIn the first case, we get $z^2=e^{\\frac{2\\pi}{3}i}$ or therefore it is the same as $\\frac{-1}{2}+\\frac{\\sqrt3}{2}i$\n\nWe then get $\\frac{\\frac32-\\frac{\\sqrt3}{2}i}{\\frac{\\sqrt3}{2}+\\frac{i}{2}}$\n$= \\frac{3-\\sqrt3*i}{\\sqrt3+i}=\\frac{(3-\\sqrt3*i)(\\sqrt3+i)}{4}=\\frac{3-4\\sqrt{3}i-3}{4}=-\\sqrt{3}i$.\n\nIn the second case, we get $z^2=e^{\\frac{4\\pi}{3}i}=-\\frac{1}{2}-\\frac{\\sqrt3}{2}i$.\nAlso, we get $\\frac{1-z^2}{z}=\\frac{3+\\sqrt3i}{1-\\sqrt3i}=\\frac{(3+\\sqrt3i)(1+\\sqrt3i)}{4}=\\frac{4\\sqrt3i}{4}=\\sqrt3i$.\n\nWe are finally done. $\\Box$. [/hide]",
"Solution_6": "We denote the affix of each uppercase letter by a lowercase letter.Then by rotation formula we have,\n\n$x=c+(b-c)\\epsilon$\n$y=a+(c-a)\\epsilon$\n$z=b+(a-b)\\epsilon$\n\nThus $n=c+\\frac{(b-c)\\epsilon}{2}$\n $n'=\\frac{(c+a)+(c-a)\\epsilon}{2}$\n $m=\\frac{(a+b)+(a-b)\\epsilon}{2}$\n $b'=\\frac{b+c}{2}$ \nIt is just calculation to see that $\\frac{m-n}{b'-n'}=i\\mathbb{R}$ for some nonzero real $\\mathbb{R}$.\n\n[u][b]Note[/b][/u]:Here $\\epsilon$ represents $cos60^{\\circ}+isin60^{\\circ}$.\n\nBye.\n\nMaths is the doctor of science...\n\nSayantan..",
"Solution_7": "Dear Mathlinkers,\nfor a more general proof, you can see\nhttp://perso.orange.fr/jl.ayme vol. 16 p. 43-44\n\nSincerely\nJean-Louis"
}
{
"Tag": [
"number theory proposed",
"number theory"
],
"Problem": "Find all prime divisors $ p$(p<1500) of $ 2^{37}\\minus{}1$\r\nI tried to find greater prime divisors,but failed",
"Solution_1": "You can chek only $ p\\equal{}1\\mod 37$ and $ p\\equal{}\\pm 1\\mod 8$.\r\nIt mean $ p\\equal{}1\\mod 296$ or $ p\\equal{}223\\mod 296$.",
"Solution_2": "hello, i have $ 2^{37}\\minus{}1\\equal{}223\\cdot616318177$.\r\nSonnhard."
}
{
"Tag": [
"number theory proposed",
"number theory"
],
"Problem": "As the title says :lol:",
"Solution_1": "This must have been posted before.\r\n\r\n[hide=\"Hint\"]Show that the product of four consecutive integers is one less than a perfect square.[/hide]",
"Solution_2": "[hide]$ x(x\\plus{}1)(x\\plus{}2)(x\\plus{}3) \\plus{} 1 \\equal{} x^4 \\plus{} 6x^3 \\plus{} 11x^2 \\plus{} 6x \\plus{} 1 \\equal{} (x^2 \\plus{} 3x \\plus{} 1)^2$[/hide]\r\n\r\nWe use the fact that if $ m$ is a perfect square then $ m\\minus{}1$ can not be a perfect square. :)",
"Solution_3": "[quote=\"B_Gasperov\"]We use the fact that if $ m$ is a perfect square then $ m \\minus{} 1$ can not be a perfect square. :)[/quote]\r\n\r\nWhat about $ m \\equal{} 1$?\r\n\r\nThe actual property you're using is that between two consecutive perfect squares there isn't another perfect square. This is true of any monotonic sequence of integers and can be used to solve several other related problems.",
"Solution_4": "everybody found my solution.",
"Solution_5": "This is part a) of a problem from the Spanish olympiad. It can be a perfect square only if one of the four terms equals zero.\r\n\r\nnow part b):\r\n\r\nProve that the product of 4 consecutive positive integers is never a perfect cube.",
"Solution_6": "[quote=\"Pul de Algodoncito\"]Prove that the product of 4 consecutive positive integers is never a perfect cube.[/quote]\r\n\r\nLet $ n(n\\plus{}3)\\equal{}2k$. Then $ (n\\plus{}1)(n\\plus{}2)\\equal{}2k\\plus{}2$. Thus \\[ P\\equal{}n(n\\plus{}1)(n\\plus{}2)(n\\plus{}3)\\equal{}4k(k\\plus{}1)\\]\r\nLet $ P\\equal{}(2t)^3$. Then $ 4k(k\\plus{}1)\\equal{}8t^3\\implies k(k\\plus{}1)\\equal{}2t^3$. So the only possible solution to this equation is $ k\\equal{}1,t\\equal{}1$, as $ (k,k\\plus{}1)\\equal{}1$. But $ k\\equal{}1$ implies $ n(n\\plus{}3)\\equal{}2$, which is impossible. Hence the conclusion.",
"Solution_7": "Let the 4 consecutive positive integers be x,x+1,x+2 and x+3\r\nTheir product = {x*(x+3})*{(x+1)*(x+2)} = (x^2+3x+1)^2 -1.\r\nSo, the product of them can't be a perfect square."
}
{
"Tag": [
"trigonometry",
"geometry",
"area of a triangle",
"Heron\\u0027s formula"
],
"Problem": "I can't do this problem... :( \r\n\r\nTwo circles have radii a,b. The distance between their centers is d. They intersect at P and Q. The distance from P to Q is?\r\n\r\nExpress your answering using a,b,and/or d.",
"Solution_1": "this seems an easy problem. let O1, O2 be the centers of the circles. draw the O1a, where it's tangent to the second circle (that would be point P), and draw O2b were it's tangent to the first one(point P). then draw the line, d. it is obvious that the angle O1PO2 is 90 :^o: and all you need to do now is to figure out the altitue of the line PH, that is orthagonal to D, and then multiply it by two. and that would be the length of PQ.",
"Solution_2": "[hide]Oops messed up the first time.\n$2a \\sin \\left( \\cos^{-1} \\left( \\frac {b^2 - a^2 - d^2}{-2ad} \\right ) \\right)$[/hide]",
"Solution_3": "Here were the choices.. Since no answer doesn't seem to give me \"variable\" answer..\r\n\r\nA) $\\frac{[d^2 - (a-b)^2]^{\\frac{1}{2}} [(a+b)^2 - d^2]^{\\frac{1}{2}}}{d}$\r\n\r\nB) $(a^2 + b^2 - d^2)^{\\frac{1}{2}}$\r\n\r\nC) $[(a-b)(a+b)]^{\\frac{1}{2}}$\r\n\r\nD) $\\frac{a+b+d}{3}$\r\n\r\nE) None of the above",
"Solution_4": "[hide]Answer: [b]A [/b]\n\n Using Heron's formula, $\\triangle PO_1O_2 =\\frac{1}{2}dh=\\frac{1}{2}\\sqrt{\\frac{a+b+d}{2}\\cdot \\frac{a+b-d}{2}\\cdot \\frac{b+d-a}{2}\\cdot \\frac{d+a-b}{2}}$[/hide]",
"Solution_5": "think of the triangle with vertices of the centers and $P$. The sides are $a,b,d$. The distance from $P$ to the base is $\\displaystyle \\frac{1}{2}$ of $PQ$.\r\nTo find the height, use Heron's formula.\r\n$\\displaystyle \\frac{1}{2}dh=\\sqrt{\\frac{a+b+d}{2}\\cdot \\frac{a+b-d}{2}\\cdot \\frac{b+d-a}{2}\\cdot \\frac{d+a-b}{2}}$\r\n\r\n$h={2}\\displaystyle \\frac{\\sqrt{\\frac{a+b+d}{2}\\cdot \\frac{a+b-d}{2}\\cdot \\frac{b+d-a}{2}\\cdot \\frac{d+a-b}{2}}}{d}$\r\n\r\n$PQ={4}\\displaystyle \\frac{\\sqrt{\\frac{a+b+d}{2}\\cdot \\frac{a+b-d}{2}\\cdot \\frac{b+d-a}{2}\\cdot \\frac{d+a-b}{2}}}{d}$\r\n\r\n$PQ=\\frac{\\sqrt{d^2 - (a-b)^2} \\sqrt{(a+b)^2 - d^2}}{d}$\r\n\r\nAnswer is $\\text {A}$"
}
{
"Tag": [
"function",
"\\/closed"
],
"Problem": "This is the second time I'm posting this, but its back again!\r\n\r\nEvery time I post in the MC forum, I get this:\r\n\r\n[code]\nSQL requests not achieved\n\nDEBUG MODE\n\nSQL Error: 1064 You have an error in your SQL syntax; check the manual that corresponds to your MySQL server version for the right syntax to use near ' 19209) GROUP BY ug.user_id' at line 7\n\nSQL Request: SELECT MAX(a.auth_value) as allowed, ug.user_id FROM phpbb_auths a, phpbb_user_group ug WHERE a.obj_type = 'f' AND a.obj_id = 132 AND a.auth_name = 'auth_watch_forum' AND a.group_id = ug.group_id AND ug.user_id IN (, 19209) GROUP BY ug.user_id\nLine : 1189\nFile : functions_post.php\n[/code]\r\n\r\nCan someone please see what's wrong?",
"Solution_1": "[quote=\"#H34N1\"]This is the second time I'm posting this, but its back again!\n\nEvery time I post in the MC forum, I get this:\n\n[code]\nSQL requests not achieved\n\nDEBUG MODE\n\nSQL Error: 1064 You have an error in your SQL syntax; check the manual that corresponds to your MySQL server version for the right syntax to use near ' 19209) GROUP BY ug.user_id' at line 7\n\nSQL Request: SELECT MAX(a.auth_value) as allowed, ug.user_id FROM phpbb_auths a, phpbb_user_group ug WHERE a.obj_type = 'f' AND a.obj_id = 132 AND a.auth_name = 'auth_watch_forum' AND a.group_id = ug.group_id AND ug.user_id IN (, 19209) GROUP BY ug.user_id\nLine : 1189\nFile : functions_post.php\n[/code]\n\nCan someone please see what's wrong?[/quote]\r\nThe post still gets made, right?",
"Solution_2": "Yes, but the messages is quite annoying.",
"Solution_3": "[quote=\"#H34N1\"]Yes, but the messages is quite annoying.[/quote]\r\n\r\nyes that has happened to me many times and i recall someone else posting this bug somewhere...",
"Solution_4": "That was me :roll: \r\n\r\nI couldn't find it so I posted it again.",
"Solution_5": "Bug reproduced, and finally fixed, thanks to Darij :)"
}
{
"Tag": [
"number theory unsolved",
"number theory"
],
"Problem": "Find all triples of positive integers $ (p,q,n)$, with $ p$ and $ q$ primes, satisfying:\r\n\r\n$ p(p\\plus{}3)\\plus{}q(q\\plus{}3)\\equal{}n(n\\plus{}3)$.",
"Solution_1": "Here is my solution.\r\nBy $ mod \\ 3$, we have $ p^2 \\plus{} q^2 \\equiv \\ n^2$. Hence, $ p$ or $ q$ is equal to $ 3$, otherwise $ n^2 \\equiv \\ 2$. Now, w.l.o.g. suppose that $ p \\equal{} 3$, so the equation becomes $ 18 \\plus{} q(q \\plus{} 3) \\equal{} n(n \\plus{} 3)$. Solving like a 2-nd degree equation in $ n$, we have the discriminant equals to $ \\Delta \\equal{} \\sqrt {4q(q \\plus{} 3) \\plus{} 81}$. Since $ n$ is a integer and $ \\Delta$ is odd, $ 4q(q \\plus{} 3) \\plus{} 81 \\equal{} (2k \\plus{} 1)^2$, where $ k$ is a nonnegative integer. Hence $ q(q \\plus{} 3) \\equal{} (k \\minus{} 4)(k \\plus{} 5)$. \r\nNow suppose that $ k \\equal{} qt \\minus{} 5$, where $ t$ is a nonnegative integer. So $ q \\plus{} 3 \\equal{} (qt \\minus{} 9)t$. So $ q \\plus{} 3\\geq qt \\minus{} 9$, so $ t\\leq \\frac{q\\plus{}12}{q}$, and it's easy to see that $ t\\leq 7$. By brute force, we can observe that $ t \\equal{} 5$ and $ p \\equal{} 2$. Substituting in the equation, the only positive integer $ n$ is $ 4$.\r\nAnalogously, the case $ k \\equal{} qt \\plus{} 4$ don't generate solutions.\r\nThe unique triple $ (p,q,n)$ is $ (2,3,4)$.",
"Solution_2": "The above solution is incorrect, as $3(3 + 3) + 7(7 + 3) = 88 = 8(8 + 3)$. Here's my attempt.\n\n\nConsidering the expression modulo 3, we note that the only quadratic residues modulo 3 are 0 and 1, thus \n$$\np(p + 3) + q(q + 3) = n(n + 3) \\iff p^2 + q^2 \\equiv n^2 \\pmod{3},\n$$\nwill have solutions only if $p^2 + q^2 \\equiv 0$ or $1 \\pmod{3}$.\n\nIf $p^2 + q^2 \\equiv 0$, then $p = q = 3$, but then there is no solutions for $n$.\n\nThis implies that one of $p$ or $q$ must be 3. WLOG let $p = 3$, then\n$$\n18 + q(q + 3) = n(n + 3) \\implies 18 = (n-q)(n + q + 3).\n$$\nChecking factor pairs of 18, we find that the only solutions are $(p, q, n) = (3, 2,4)$ and $(3, 7, 8)$,\nso all of the solutions to the equation are\n$$\n(p, q, n) \\in \\{(2, 3, 4), \\; (3, 2, 4),\\; (3, 7, 8), \\; (7, 3, 8)\\}.\n$$",
"Solution_3": "As people above me have proven, one of the primes p or q equals 3. WLOG q=3\nThen we expand and get 18+q\u00b2-n\u00b2=3(n-q) using diffrence of squares and changing sides we get 18=(n-q)(3+q+n) so we have n>q and because q\u22652 we have 3+q+n\u22658 and 3+q+n divides 18 so either 3+q+n=9 and n-q=2 or 3+q+n=18 and n-q=1"
}
{
"Tag": [],
"Problem": "I had a simple conceptual doubt once and forgot all about that. Now that I recall it could someone help me out...\r\n\r\nThere is a block placed on a rough horizontal surface such that it just touches a wall perpendicular to the floor. Now if a slight increasing external force is applied pushing it against the wall then what are the normal forces (from the wall) and the frictional forces.\r\n\r\nI mean suppose the limiting friction is 10N. And a force of 5N is applied then is the entire 5N opposed by friction or the normal force of the wall. If not then how does the 5N divide among the two opposing forces ?\r\n\r\nI am sorry if my doubt is really stupid :-) .But please help..",
"Solution_1": "The 5N countering the force pushing against the wall is the normal force, but I don't see where you're getting the frictional force from.\r\n\r\nConstructing the free body diagram for this system, if we suppose the walls are below and to the right of the block, you should get four forces as follows:\r\n-Pushing force to the right\r\n-Weight acting downwards\r\n-Normal force acting upwards\r\n-Normal force acting left\r\n\r\nThe left normal force counters the right force and the upwards normal force counters the downwards weight.",
"Solution_2": "Thanks its clear now...\r\nThere is no friction when there is no tendancy to move"
}
{
"Tag": [
"LaTeX",
"irrational number"
],
"Problem": "How can I solve $\\sqrt{164}$ without using a calculator?",
"Solution_1": "[quote=\"blueshark\"]How can I solve sqrt{164} without using a calculator?[/quote]To make that appear as $\\sqrt{164}$, just type \\sqrt{164} and type dollar signs before and after it. If you're going to be around here for a while you should learn $\\LaTeX$. It's very easy.\r\n\r\nAnyway, I'm not sure what you mean when you say you want to \"solve $\\sqrt{164}$.\" If you want to find it's decimal expansion, look [url=http://www.artofproblemsolving.com/Forum/viewtopic.php?t=48990]here[/url]. It's an irrational number, so it will have an infinite nonrepeating decimal expansion, so you won't be able to actually write $\\sqrt{164}$ completely as a decimal.",
"Solution_2": "By solve,do u mean like till infinite digits??cause we can write sqroot164 as 2*sqroot41 and then find sqroot 41 by the normal method(as taught in school in around 8th grade).We can find the appprox value by this method.",
"Solution_3": "Thanks for your reply. What I meant to ask is: What is\r\n$\\sqrt{164}$ correct to the nearest integer? How can I find the answer without using a calculator?\r\n\r\nFor example, I know how to solve problems like $\\sqrt{169}$ which equals 13 because 13 times 13 = 169, right? The same for applies for the sqaure root of 36, 25, 81, 9, 4, 16, 100, etc. I recall my teacher saying that numbers like 4, 9, 16, 25, 36, 81, and 100 are called perfect squares, right? \r\n\r\nHow do I answer the question above without using a calculator?",
"Solution_4": "something like this ?\r\n\r\n$\\sqrt{144}<\\sqrt{164}<\\sqrt{169}$ so $12<\\sqrt{164}<13$ :huh: \r\n\r\nThis is basically through guessing the approximate like what is the nearest integer for\r\n\r\n$\\sqrt{3724}$ ? So you know $\\sqrt{3600}=60$ so the integer should be starting with $6$ . Then just work out some number $61^2=3721$ , $62^2=3844$ ,$63^2=3969$ and you will get the nearest integer to be $61$ :)",
"Solution_5": "i'm sorry for the horrifying diagram,but thats the method i follow for findin square roots.if u multiply 6.4 by 2,u get 12.8 and sqroot 164 is 12.806.....",
"Solution_6": "[hide]\nsince 164 is between $12^2$ and $13^2$ the answer is betwen 12 and 13. 164 is closer to 169 than 144 so the answer is definetly bigger than 12.5. Seeing that 164 is only 5 off from 169 and 20 off from 144 we can estimate that the answer is 12.8 or 12.9. Trying 12.8 i get 163.84 so 12.8 is close. Trying 12.9 i get 166.41 so the answer is approximatley 12.8.[/hide]",
"Solution_7": "If you want to find the value of $\\sqrt{164}$..\r\n\r\n[hide]Find the nearest perfect square, 169, and take the square root of that, which is 13. Divide 164 by 13, to get (I like to cheat with calculators) 12.62. Take the average of that and 13 to get 12.81. Now divide 164 by 12.81 to get 12.805, take the average of 12.81 and 12.805 to get about 12.806. Continue until you get to the desired place. [/hide]",
"Solution_8": "[hide]\n164 is in between 12^2=144 and 13^2=169,\n164 is closer to 169 then 144, so it will be more than 12.5.\nTry 12.8, you get 163.84\nTry 12.9, you get 166.41\n163.84 is closer to 164 than 166.41\n$\\sqrt{164} \\approx \\framebox{12.8}$[/hide]",
"Solution_9": "I hope you mean 12.8 in boxes :) \r\n\r\nThat's a method too. There are many.",
"Solution_10": "http://www.mathpath.org/Algor/algor.square.root.htm",
"Solution_11": "I want to thank all the tutors who took time to explain my question.",
"Solution_12": "$\\sqrt{144}$<$\\sqrt{164}$<$\\sqrt{169}$\r\n\r\nSo, 12<$\\sqrt{164}$<13",
"Solution_13": "well it is my first post in this sub-forum \r\n$\\sqrt{164} \\approx \\frac{333}{26}$ :D :D \r\nI use this formula \\[ \\sqrt{x} \\approx \\frac{2x + p}{2 \\sqrt{x+p}} \\]\r\ni choose $p=5$.\r\nwant a proof??? :)",
"Solution_14": "I don't know if this was posted earlier, but you could always use the binomial theorem. ;)"
}
{
"Tag": [
"algebra unsolved",
"algebra",
"combinatorics",
"Combinatorial sum"
],
"Problem": "Find The Closed form of:\r\n\r\n$\\sum_{k=0,n}(-1)^k(n-k)!(n+k)!$",
"Solution_1": "It is: $(2n)!(1-\\frac{1}{2(n+1)})$ if I didn't miscalculated.\r\n\r\nI used a little changed Australian problem: $\\sum _0 ^{2n} \\frac{(-1)^k}{C_{2n} ^k}=2-\\frac{1}{n+1}$",
"Solution_2": "which year's TST was this from?",
"Solution_3": "[quote=\"Megus\"]It is: $(2n)!(1-\\frac{1}{2(n+1)})$ if I didn't miscalculated.\n\nI used a little changed Australian problem: $\\sum _0 ^{2n} \\frac{(-1)^k}{C_{2n} ^k}=2-\\frac{1}{n+1}$[/quote]\r\nhow to prove the australian problem?",
"Solution_4": "you can use induction.\r\ni'll try to write a detailed solution in a moment",
"Solution_5": "seems that i'm wrong :(",
"Solution_6": "[quote=\"perfect_radio\"]\nFor $n=1$ we have $\\frac{1}{C_{2}^0-C_{2}^1+C_{2}^2}=2-\\frac{1}{2}$\n$\\sum _{k=0} ^{2n} \\frac{(-1)^k}{C_{2n} ^k}=2-\\frac{1}{n+1}$[/quote]\r\n\r\nThere, I think the [ was supposed to be a { ;)",
"Solution_7": "Where can I find a complete solution?",
"Solution_8": "To prove australian problem first prove that:\r\n\r\n$\\frac{1}{C _{2n}^k}=\\frac{2n+1}{2n+2}(\\frac{1}{C_{2n+1}^{k}}+\\frac{1}{C_{2n+1}^{k+1}})$",
"Solution_9": "i just checked.. it's from the 2001 TST. you can find them at http://www.unl.edu/amc/a-activities/a7-problems/problemarchive.html, though there aren't any solutions posted."
}
{
"Tag": [
"search",
"function"
],
"Problem": "Heloo everyone.\r\nI will be very pleased if s.o. of you can write me links or problems,which have proofed with Stewart's theorem.\r\nThank you.",
"Solution_1": "Use the Search function to get some problems involving Stewart's theorem. [url]http://www.mathlinks.ro/Forum/search.php[/url]\r\nAnd also it not regards Middle School, of course. :!:",
"Solution_2": "[quote=\"Andreas\"]Use the Search function to get some problems involving Stewart's theorem. [url]http://www.mathlinks.ro/Forum/search.php[/url]\nAnd also it not regards Middle School, of course. :!:[/quote]\r\nThank you"
}
{
"Tag": [
"limit"
],
"Problem": "\u039b\u03bf\u03b9\u03c0\u03bf\u03bd \u03bd\u03b1 \u03bc\u03b9\u03b1 \u03bb\u03b9\u03b3\u03bf \u03c0\u03b5\u03c1\u03b9\u03b5\u03c1\u03b3\u03b7 \u03b1\u03c3\u03ba\u03b7\u03c3\u03b7.\u039a\u03b1\u03bb\u03bf\u03ba\u03b1\u03b9\u03c1\u03b9\u03bd\u03b7 \u03c0\u03b5\u03b9\u03c4\u03b5....\r\n\u03a4\u03b7\u03bd \u03b1\u03c6\u03b9\u03b5\u03c1\u03c9\u03bd\u03c9 \u03c3\u03b5 \u03bf\u03bb\u03bf\u03c5\u03c2 .\r\n\r\n\u0391\u03bd $a,b,c,d \\in \\mathbb{R}$ \u03bc\u03b5 \r\n\r\n$a+b+c+d<-1$\r\n$a-b+c-d<-1$\r\n$\\mid 4a+2c \\mid=\\mid 3b+d \\mid$\r\n$\\mid b \\mid \\leq 1$\r\n\r\n\u03bd\u03b4\u03bf $ac\\leq \\frac{3}{8}$\r\n\r\n :)",
"Solution_1": "\u03a9\u03c1\u03b1\u03af\u03b1 \u03b7 \u03ac\u03c3\u03ba\u03b7\u03c3\u03b7 Socrates :) . \u03a0\u03bf\u03cd \u03c4\u03b7\u03bd \u03b2\u03c1\u03ae\u03ba\u03b5\u03c2 ??\r\n\u039d\u03b1 \u03bc\u03b9\u03b1 \u03c0\u03c1\u03bf\u03c4\u03b5\u03b9\u03bd\u03cc\u03bc\u03b5\u03bd\u03b7 \u03bb\u03cd\u03c3\u03b7 : \r\n\r\n[hide=\"\u039b\u03cd\u03c3\u03b7\"]\u0398\u03b5\u03c9\u03c1\u03bf\u03cd\u03bc\u03b5 \u03c4\u03bf \u03c0\u03bf\u03bb\u03c5\u03ce\u03bd\u03c5\u03bc\u03bf $p(x) = a x^{4} +b x^{3} + c x^{2} +d x +1$.\n\n\u0388\u03c7\u03bf\u03c5\u03bc\u03b5 $p(1)=a+b+c+d +1<0$ \u03ba\u03b1\u03b9 $p(-1)=a-b+c-d +1<0$ .\n\n\u0391\u03c6\u03bf\u03cd $p(0)=1>0$ \u03c4\u03bf \u03c0\u03bf\u03bb\u03c5\u03ce\u03bd\u03c5\u03bc\u03bf \u03ad\u03c7\u03b5\u03b9 \u03b4\u03cd\u03bf \u03c1\u03af\u03b6\u03b5\u03c2 $\\rho_{1} \\in (-1,0)$ \u03ba\u03b1\u03b9 $\\rho_{2} \\in (0,1)$.\n\n[u][b]\u03a0\u03b5\u03c1\u03af\u03c0\u03c4\u03c9\u03c3\u03b7 1[/b][/u]\n\n\u0386\u03bd $\\boxed{a>0}$ \u03b1\u03c6\u03bf\u03cd $\\lim_{x \\to + \\infty}p(x)=\\lim_{x \\to -\\infty}p(x)=+ \\infty$ \u03ba\u03b1\u03b9 $p(1), \\, p(-1) \\, <0$ \n\n\u03c4\u03bf \u03c0\u03bf\u03bb\u03c5\u03ce\u03bd\u03c5\u03bc\u03bf \u03ad\u03c7\u03b5\u03b9 \u03ac\u03bb\u03bb\u03b5\u03c2 \u03b4\u03cd\u03bf \u03c1\u03af\u03b6\u03b5\u03c2 $\\rho_{3} \\in (-\\infty,-1)$ \u03ba\u03b1\u03b9 $\\rho_{4} \\in (1, \\infty)$ .\n\n[u][b]\u03a0\u03b5\u03c1\u03af\u03c0\u03c4\u03c9\u03c3\u03b7 2[/b][/u]\n\n$\\boxed{a<0}$\n\n\u0388\u03c0\u03af\u03c3\u03b7\u03c2 $p'(x)=4ax^{3}+3bx^{2}+2cx+d$ \u03ac\u03c1\u03b1 \u03b1\u03c0\u03cc \u03c4\u03b7\u03bd \u03c3\u03c5\u03bd\u03b8\u03ae\u03ba\u03b7\n\n$\\\\ \\mid 4a+2c \\mid=\\mid 3b+d \\mid$ \u03c0\u03b1\u03af\u03c1\u03bd\u03bf\u03c5\u03bc\u03b5 $p'(1)=0$ \u03ae $p'(-1)=0$ .\n\n\u03a5\u03c0\u03bf\u03b8\u03ad\u03c4\u03bf\u03c5\u03bc\u03b5 \u03cc\u03c4\u03b9 $p'(1)=4a+3b+2c+d=0$. \n\n\u0391\u03c5\u03c4\u03cc \u03bc\u03c0\u03bf\u03c1\u03b5\u03af \u03bd\u03b1 \u03c3\u03c5\u03bc\u03b2\u03b5\u03af \u03cc\u03c4\u03b1\u03bd $(4a+2c)(3b+d) \\leq 0$ .\n\n$\\bullet$ \u0386\u03bd $(4a+2c)(3b+d) =0$ \u03c4\u03cc\u03c4\u03b5 $p'(-1)$ \u03c3\u03c5\u03bd\u03b5\u03c0\u03ce\u03c2 \u03c4\u03bf $p'(x)$ \u03ad\u03c7\u03b5\u03b9 \u03c4\u03c1\u03b5\u03b9\u03c2 \u03c1\u03af\u03b6\u03b5\u03c2.\n\n\n$\\bullet$ \u0391\u03bd $4a+2c <0$ \u03ba\u03b1\u03b9 $3b+d>0$ \u03c4\u03cc\u03c4\u03b5 $p'(-1)=(3b+d)-(4a+2c) >0$.\n\u03a3\u03b5 \u03b1\u03c5\u03c4\u03ae \u03c4\u03b7\u03bd \u03c0\u03b5\u03c1\u03af\u03c0\u03c4\u03c9\u03c3\u03b7 \u03b1\u03bd \u03c5\u03c0\u03bf\u03b8\u03ad\u03c3\u03bf\u03c5\u03bc\u03b5 \u03cc\u03c4\u03b9 $p'(x)>0$ \u03b3\u03b9\u03b1 $x \\in (- \\infty ,1)$ \u03ba\u03b1\u03c4\u03b1\u03bb\u03ae\u03b3\u03bf\u03c5\u03bc\u03b5 \u03c3\u03b5 \n\u03ac\u03c4\u03bf\u03c0\u03bf \u03b3\u03b9\u03b1\u03c4\u03af \u03c4\u03bf \u03c0\u03bf\u03bb\u03c5\u03ce\u03bd\u03c5\u03bc\u03bf \u03ad\u03c7\u03b5\u03b9 \u03c1\u03af\u03b6\u03b1 \u03c3\u03c4\u03bf $(0,1)$ .\n\u0386\u03c1\u03b1 \u03ba\u03ac\u03c0\u03bf\u03c5 \u03b1\u03bb\u03bb\u03ac\u03b6\u03b5\u03b9 \u03c0\u03c1\u03cc\u03c3\u03b7\u03bc\u03bf \u03ba\u03b1\u03b9 \u03ad\u03c4\u03c3\u03b9 \u03c0\u03b1\u03af\u03c1\u03bd\u03bf\u03c5\u03bc\u03b5 \u03c0\u03ac\u03bb\u03b9 3 \u03c1\u03af\u03b6\u03b5\u03c2 .\n\n\n$\\bullet$ \u0391\u03bd $4a+2c >0$ \u03ba\u03b1\u03b9 $3b+d<0$ \u03c4\u03cc\u03c4\u03b5 $p'(-1)<0$ .\n\u03a4\u03ce\u03c1\u03b1 \u03b1\u03c6\u03bf\u03cd $\\lim_{x \\to - \\infty}p'(x)=+ \\infty$ \u03c4\u03bf $p'(x)$ \u03ad\u03c7\u03b5\u03b9 \u03c1\u03af\u03b6\u03b1 $\\xi \\in (- \\infty ,-1)$ \u03ac\u03c1\u03b1 \u03ad\u03c7\u03b5\u03b9 3 \u03c1\u03af\u03b6\u03b5\u03c2 .\n[u][b]\n\u03a0\u03b5\u03c1\u03af\u03c0\u03c4\u03c9\u03c3\u03b7 3[/b][/u]\n\n$\\\\ a=0$ :D \n\n\u0391\u03c0\u03cc \u03cc\u03bb\u03b1 \u03c4\u03b1 \u03c0\u03b1\u03c1\u03b1\u03c0\u03ac\u03bd\u03c9 \u03c4\u03bf $p''(x)=12ax^{2}+6bx+2c$ \u03ad\u03c7\u03b5\u03b9 2 \u03c1\u03af\u03b6\u03b5\u03c2\n\u03ac\u03c1\u03b1 $D = 36b^{2}-96ac \\geq 0 \\Rightarrow \\frac{3}{8} \\cdot b^{2} \\geq ac$\n\n\u03ba\u03b1\u03b9 \u03b1\u03c6\u03bf\u03cd $|b| \\leq 1$ \u03c0\u03b1\u03af\u03c1\u03bd\u03bf\u03c5\u03bc\u03b5 $\\boxed{ac \\leq \\frac{3}{8}}$ .[/hide]\r\n\r\n\u03a0\u03ae\u03c1\u03b1 \u03c0\u03bf\u03bb\u03bb\u03ad\u03c2 \u03c0\u03b5\u03c1\u03b9\u03c0\u03c4\u03ce\u03c3\u03b5\u03b9\u03c2 \u03ba\u03b1\u03b9 \u03b1\u03c5\u03c4\u03cc \u03b4\u03b5\u03bd \u03bc\u03bf\u03c5 \u03b1\u03c1\u03ad\u03c3\u03b5\u03b9. \r\n\u0398\u03b1 \u03c0\u03c1\u03bf\u03c3\u03c0\u03b1\u03b8\u03ae\u03c3\u03c9 \u03b3\u03b9\u03b1 \u03ba\u03ac\u03c4\u03b9 \u03c0\u03b9\u03bf \u03ba\u03bf\u03bc\u03c8\u03cc \u03b1\u03cd\u03c1\u03b9\u03bf . :)"
}
{
"Tag": [
"algebra",
"polynomial",
"vector",
"complex numbers",
"superior algebra",
"superior algebra unsolved"
],
"Problem": "1. Let $\\alpha, \\beta$ be complex numbers of degree 3 over $\\mathbb{Q}$. Determine the possibilities for $[\\mathbb{Q}(\\alpha, \\beta) : \\mathbb{Q}]$.\r\n\r\n2. Let $\\alpha, \\beta$ be complex roots of irreducible polynomials $f(x), g(x) \\in \\mathbb{Q}[x]$. Prove that $f(x)$ is irreducible in $\\mathbb{Q}[\\alpha]$ iff $g(x)$ is irreducible in $\\mathbb{Q}[\\beta]$.",
"Solution_1": "1. Suppose $[\\mathbb{Q}(\\alpha): \\mathbb{Q}]=3$ and $[\\mathbb{Q}(\\beta): \\mathbb{Q}]=3$. Then $3~\\leq[\\mathbb{Q}(\\alpha,\\beta): \\mathbb{Q}]\\leq~9$. \r\nIf $\\mathbb{Q}(\\alpha)=\\mathbb{Q}(\\beta)$ then $[\\mathbb{Q}(\\alpha,\\beta): \\mathbb{Q}]=3$. If no common basis elements of $\\mathbb{Q}(\\alpha)$ and $\\mathbb{Q}(\\beta)$ as vector spaces over $\\mathbb{Q}$, then $[\\mathbb{Q}(\\alpha,\\beta): \\mathbb{Q}]=9$.",
"Solution_2": "[quote=\"abdul\"]If no common basis elements of $\\mathbb{Q}(\\alpha)$ and $\\mathbb{Q}(\\beta)$ as vector spaces over $\\mathbb{Q}$, then $[\\mathbb{Q}(\\alpha,\\beta): \\mathbb{Q}]=9$.[/quote]\r\nThis is not true, you can still get degree $6$.\r\n\r\n2. is a matter of degree calculation, too.",
"Solution_3": "When does this equality $[\\mathbb{Q}(\\alpha,\\beta): \\mathbb{Q}]=9$ hold?",
"Solution_4": "$\\alpha = \\sqrt[3]{2}, \\beta = \\sqrt[3]{3}$.",
"Solution_5": "This what i think too when i made $9$ as upper bound.\r\nBut how can still get degree $6$ if no common basis elements of $\\mathbb{Q}(\\alpha)$ and $\\mathbb{Q}(\\beta)$?\r\n\r\nI just read about field extensions for first time some hours ago..",
"Solution_6": "You can even have $\\mathbb{Q}(\\alpha)=\\mathbb{Q}(\\beta)$ but disjoint bases, simply as bases can be choosen quite random (you mean something else than bases I think).\r\nOne example for degree $6$: $\\alpha = \\sqrt[3]{2}$ and $\\beta = \\zeta_{3}\\sqrt[3]{2}$ with $\\zeta_{3}^{3}=1 , \\zeta_{3}\\neq 1$.",
"Solution_7": "Thank you, ZetaX."
}
{
"Tag": [
"inequalities proposed",
"inequalities"
],
"Problem": "This is MIC round 3 and its solution... We are looking for your ideas. :)\r\n\r\nRegards\r\nVo Quoc Ba Can",
"Solution_1": "[quote=\"toanhocmuonmau\"]This is MIC round 3 and its solution... We are looking for your ideas. :)\n\nRegards\nVo Quoc Ba Can[/quote]\r\nOne of it was from here:http://www.aoshoo.com/bbs1/dispbbs.asp?boardid=48&Id=14604&page=2 :)"
}
{
"Tag": [],
"Problem": "A recursive sequence has terms in the form $ a_n/b_n$ and is defined such that all even numbered terms are the reciprocal of the immediately preceding term and in all odd numbered terms, $ a_n$ and $ b_n$ are equal to $ a_{n \\minus{} 2} \\plus{} 1$ and $ b_{n \\minus{} 2} \\plus{} 1$ respectively. If $ t_1 \\equal{} 2/3$ what is: \r\n\r\n$ (a_{2005} \\plus{} a_{2006} \\plus{} a_{2007} \\plus{} a_{2008})/(b_{2005} \\plus{} b_{2006} \\plus{} b_{2007} \\plus{} b_{2008})$",
"Solution_1": "[hide=\"Solution\"]\n\nNotice that\n\n$ a_{2k} \\equal{} a_{2k \\minus{} 1} \\plus{} 1$ and\n$ a_{2k \\plus{} 1} \\equal{} a_{2k}$\nfor any integer k.\n\nAlso notice that\n\n$ b_{2j} \\equal{} b_{2j \\minus{} 1} \\minus{} 1$ and\n$ b_{2j \\plus{} 1} \\equal{} b_{2j} \\plus{} 2$\n\nWe can turn the recursive definition of sequence $ \\{t\\}$ into a... a... (the word is slipping from my mind) definition where you can find the term with the index.\n\n$ a_x \\equal{} (x \\plus{} 3)/2$ if x is odd\n$ a_x \\equal{} (x \\plus{} 4)/2$ if x is even\n$ b_x \\equal{} (x \\plus{} 5)/2$ if x is odd\n$ b_x \\equal{} (x \\plus{} 2)/2$ if x is even\n\nTherefore,\n\n$ (a_{2005} \\plus{} a_{2006} \\plus{} a_{2007} \\plus{} a_{2008})/(b_{2005} \\plus{} b_{2006} \\plus{} b_{2007} \\plus{} b_{2008})$\n\n$ \\equal{} (1004 \\plus{} 1005 \\plus{} 1005 \\plus{} 1006)/(1005 \\plus{} 1004 \\plus{} 1006 \\plus{} 1005)$\n\n$ \\equal{} 1$\n\nWe can generalize this. For indices $ 4n \\plus{} 1, 4n \\plus{} 2, 4n \\plus{} 3, 4n \\plus{} 4$ (a is an integer),\n\n$ (a_{4n \\plus{} 1} \\plus{} a_{4n \\plus{} 2} \\plus{} a_{4n \\plus{} 3} \\plus{} a_{4n \\plus{} 4})/(b_{4n \\plus{} 1} \\plus{} b_{4n \\plus{} 2} \\plus{} b_{4n \\plus{} 3} \\plus{} b_{4n \\plus{} 4})$\n\n$ \\equal{} [(2n \\plus{} 2) \\plus{} (2n \\plus{} 3) \\plus{} (2n \\plus{} 3) \\plus{} (2n \\plus{} 4)]/[(2n \\plus{} 3) \\plus{} (2n \\plus{} 2) \\plus{} (2n \\plus{} 4) \\plus{} (2n \\plus{} 3)]$\n\n$ \\equal{} 1$\n\n[/hide]",
"Solution_2": "[quote=\"Aryth\"][...] (the word is slipping from my mind) [...][/quote]\r\n\r\nExplicit? :wink:",
"Solution_3": "Good call :P"
}
{
"Tag": [
"function",
"inequalities proposed",
"inequalities"
],
"Problem": "Let $ n \\geq 3$ be an odd integer. Determine the maximum value of\r\n\\[ \\sqrt{|x_{1}\\minus{}x_{2}|}\\plus{}\\sqrt{|x_{2}\\minus{}x_{3}|}\\plus{}\\ldots\\plus{}\\sqrt{|x_{n\\minus{}1}\\minus{}x_{n}|}\\plus{}\\sqrt{|x_{n}\\minus{}x_{1}|},\\]\r\nwhere $ x_{i}$ are positive real numbers from the interval $ [0,1]$.",
"Solution_1": "[hide=\"no proof\"]\n$ n \\minus{} 2 \\plus{} \\sqrt {2}$. My proof is inelegant.\n[/hide]",
"Solution_2": "[quote=\"JoeBlow\"][hide=\"no proof\"]\n$ n \\minus{} 2 \\plus{} \\sqrt {2}$. My proof is inelegant.\n[/hide][/quote]\r\nAfter solving it I understand what you mean! I solved it making extensive use of mixing variables, to reduce it to maximizing a function defined for each partition of n.",
"Solution_3": "We have a continuos function on a compact set $ [0,1]^n$, hence there is an optimal point $ (x_1,...,x_n)$.\r\nNote now that\r\n0) impossible to have $ x_{i \\minus{} 1} \\equal{} x_{i} \\equal{} x_{i \\plus{} 1}$;\r\n1) if $ x_i\\leq x_{i \\minus{} 1}$ and $ x_i\\leq x_{i \\plus{} 1}$, then $ x_i \\equal{} 0$;\r\n2) if $ x_i\\geq x_{i \\minus{} 1}$ and $ x_i\\geq x_{i \\plus{} 1}$, then $ x_i \\equal{} 1$;\r\n3) if $ x_{i \\plus{} 1}\\leq x_i\\leq x_{i \\minus{} 1}$ or $ x_{i \\minus{} 1}\\leq x_i\\leq x_{i \\plus{} 1}$, then $ x_i \\equal{} \\frac {x_{i \\minus{} 1} \\plus{} x_{i \\plus{} 1}}{2}$.\r\nIt follows that $ (x_1,...,x_n)$ looks like\r\n\\[ (0,\\frac {1}{k_1},\\frac2{k_1},...,1,\\frac {k_2 \\minus{} 1}{k_2},...,\\frac2{k_2},\\frac1{k_2},0,\\frac1{k_3},...,\\frac1{k_l}),\r\n\\]\r\nwhere $ k_1$, $ k_2$, ..., $ k_l$ are natural numbers, $ k_1 \\plus{} k_2 \\plus{} ... \\plus{} k_l \\equal{} n$, $ l$ is even clearly. Then the function is this point equals\r\n\\[ S \\equal{} \\sqrt {k_1} \\plus{} \\sqrt {k_2} \\plus{} ...\\sqrt {k_l}.\r\n\\]\r\nUsing the fact that $ l$ is even and $ \\sqrt {k} < \\sqrt {k \\minus{} 1} \\plus{} 1$ we conclude that maximal possible value of $ S$ is $ n \\minus{} 2 \\plus{} \\sqrt 2$ ($ l \\equal{} n \\minus{} 1$, $ k_1 \\equal{} k_2 \\equal{} ... \\equal{} k_{l \\minus{} 1} \\equal{} 1$, $ k_l \\equal{} 2$ in this case).\r\n\r\nIt is not a real olympiad problem in sense of inner beauty.",
"Solution_4": "since $ n$ is odd, there must be an $ i$ such that both $ x_i$ and $ x_{i \\plus{} 1}$ are both belong to $ [0,\\frac {1}{2}]$ or $ [\\frac {1}{2},1]$. without loss of generality let $ x_1\\leq x_2$ and $ x_1$, $ x_2$ belong to $ [0,\\frac {1}{2}]$. We can prove that $ \\sqrt {x_2 \\minus{} x_1} \\plus{} \\sqrt {Ix_3 \\minus{} x_2I}\\leq \\sqrt {2}$. If $ x_3 > x_2$, $ \\sqrt {x_2 \\minus{} x_1} \\plus{} \\sqrt {x_3 \\minus{} x_2}\\leq 2\\cdot \\sqrt {\\frac {x_3 \\minus{} x_1}{2}}\\leq \\sqrt {2}$; else $ x_1,x_2,x_3$ are all belong to $ [0,\\frac {1}{2}]$. Hence, $ \\sqrt {x_2 \\minus{} x_1} \\plus{} \\sqrt {Ix_3 \\minus{} x_2I}\\leq \\sqrt \\frac {1}{2} \\plus{} \\sqrt \\frac {1}{2}$. Also all of the other terms of the sum are less then or equal to $ 1$. summing them gives the desired result.\r\nexample is $ (0,\\frac {1}{2},1,0,1,\\ldots ,1)$\r\nnote: all the indices are considered in modulo $ n$",
"Solution_5": "We can assume there exists some $ i$ for which $ x_i \\leq x_{i\\plus{}1} \\leq x_{i\\plus{}2}$ because if it didnt exist: we can WLOG suppose $ x_1\\equal{}max{x_i}$. Now $ x_1 \\geq x_2$. It follows that $ x_2 \\leq x_3$ because if not our assumption would be faulty. Now we continue, it would be: $ x_1 \\geq x_2 \\leq x_3 \\geq x_4 .... \\leq x_n \\geq x_1$, because n is odd. It's a contradiction that $ x_1\\equal{}max{x_i}$. So there exists $ i$ such that $ x_i \\leq x_{i\\plus{}1} \\leq x_{i\\plus{}2}$. Now we can use AM-QM: $ \\sqrt{_x{i\\plus{}2}\\minus{}x_{i\\plus{}1}}\\plus{}\\sqrt{x_{i\\plus{}1}\\minus{}x_i} \\leq \\sqrt{2}\\sqrt{x_{i\\plus{}2}\\minus{}x_i}$. Now, maximum is attained if all roots are equal to $ 1$. Now it easy to check this is attainable, if $ i$ is even, then $ (x_1,x_2,...,x_i,x_{i\\plus{}1},x_{i\\plus{}2},...,x_n)\\equal{}(0,1,...,1,\\frac{1}{2},0,...,1)$. Similarly if $ i$ is odd. The maximum is $ n\\minus{}2\\plus{}\\sqrt{2}$.",
"Solution_6": "I think that this is pretty similar to IMO 2021/2. Here we can use shifting to let the biggest number be $1$ or the smallest to be $0$, and then induction will probably finish. "
}
{
"Tag": [
"linear algebra",
"matrix"
],
"Problem": "$ A*\\begin{pmatrix}\r\n 2&4\\\\\r\n 1&3\r\n \\end{pmatrix}\\equal{}\\begin{pmatrix}\r\n 2&4\\\\\r\n 1&3\r\n \\end{pmatrix}*\\begin{pmatrix}\r\n 1&1\\\\\r\n 11&5\r\n \\end{pmatrix}$\r\n$ A\\equal{}\\begin{pmatrix}\r\n 1&1\\\\\r\n 11&5\r\n \\end{pmatrix}$where A is 2x2 matrix",
"Solution_1": "No. it's not true"
}
{
"Tag": [
"algebra unsolved",
"algebra"
],
"Problem": "Find all functions $ f : \\mathbb{R}\\to\\mathbb{R}$ such that for any $ x,y\\in \\mathbb R$,\n\\[ f(x^2 - y^2) = (x - y)\\cdot [f(x) + f(y)].\\]",
"Solution_1": "[quote=\"cnyd\"]$ f: R\\rightarrow R$ $ ,$ $ \\forall x,y\\in\\ R$ $ f(x^2 \\minus{} y^2) \\equal{} (x \\minus{} y).[f(x) \\plus{} f(y)]$[/quote]\r\n\r\n[hide=\"My solution\"]\nLet $ P(x,y)$ the assertion $ f(x^2\\minus{}y^2)\\equal{}(x\\minus{}y)(f(x)\\plus{}f(y))$\n\n$ P(x,x)$ $ \\implies$ $ f(0)\\equal{}0$\n$ P(x,0)$ $ \\implies$ $ f(x^2)\\equal{}xf(x)$\n$ P(\\minus{}x,0)$ $ \\implies$ $ f(x^2)\\equal{}\\minus{}xf(\\minus{}x)$ $ \\implies$ $ f(\\minus{}x)\\equal{}\\minus{}f(x)$\n$ P(x,\\minus{}y)$ $ \\implies$ $ f(x^2\\minus{}y^2)\\equal{}(x\\plus{}y)(f(x)\\minus{}f(y))$\n\nSo $ (x\\minus{}y)(f(x)\\plus{}f(y))\\equal{}(x\\plus{}y)(f(x)\\minus{}f(y))$ and so $ xf(y)\\equal{}yf(x)$ and so $ \\frac{f(x)}{x}\\equal{}\\frac{f(y)}{y}$ $ \\forall x,y\\neq 0$\n\nSo $ f(x)\\equal{}ax$ $ \\forall x$ and it is easy to check back that this solution fits the initial equation.\n[/hide]",
"Solution_2": "$ \\frac{f(x)}{x}\\equal{}\\frac{f(y)}{y}$ $ \\implies$ $ f(x) \\equal{} ax$ Could you explain better?",
"Solution_3": "[quote=\"cnyd\"]$ \\frac {f(x)}{x} \\equal{} \\frac {f(y)}{y}$ $ \\implies$ $ f(x) \\equal{} ax$ Could you explain better?[/quote]\r\n\r\n$ \\forall x\\neq 0$ $ \\frac{f(x)}{x}\\equal{}\\frac{f(1)}{1}$ and so $ f(x)\\equal{}f(1)x$ $ \\forall x$",
"Solution_4": "aah thanks :lol: :D"
}
{
"Tag": [
"geometry",
"geometric transformation"
],
"Problem": "A particle moves through the first quadrant of the figure as follows: during the first minute it moves from the origin to (1,0). Then, it continues to follow the pattern indicated in the figure, going back and forth between the positive x and y axes, moving one unit of distance parallel to an axis in one minute. Which point will the particle reach after exactly 2 hours? \r\n\r\n\r\n[img]http://www.kangurusa.com/clark/pdb/kangur_2005_0708/output/kangur_2005_0708_30.png[/img]",
"Solution_1": "Um I'm sorry I can't really understand how the pattern goes, can you clarify that?",
"Solution_2": "I think I'm misreading the diagram or the question somehow, but I'll try anyway.\r\n\r\n[hide=\"Solution\"]2 hours is 120 minutes, so 120 moves are made. The pattern tells us that the particle essentially moves from $ (a,b)$ to $ (a\\plus{}4,b\\plus{}2)$ every 22 minutes (if I read your diagram correctly...)\n\n$ 120\\equal{}5\\cdot 22\\plus{}10$, so we will make 5 of these translations from $ (0,0)$. After 110 minutes, the particle is at $ (4\\cdot 5,2\\cdot 5)\\equal{}(20,10)$.\n\nNow we have 10 moves left. Checking the diagram, if we start at $ (a,b)$, we move to $ (a\\plus{}3,b\\plus{}1)$ in 10 steps, so we end up at $ \\boxed{(23,11)}$.[/hide]"
}
{
"Tag": [
"function",
"calculus",
"calculus computations"
],
"Problem": "Prove that a continuous function $ f: M \\rightarrow \\mathbb{R}$, all of whose values are integers, is constant provided that $ M$ is connected.\r\n\r\nIn trying to start this problem as I started looking at $ f^{ \\minus{} 1}(i)$ where $ i \\in \\mathbb{Z}$, is $ f^{ \\minus{} 1}(i)$ open and why?",
"Solution_1": "$ f^{\\minus{}1}(\\{i\\})\\equal{}f^{\\minus{}1}(i\\minus{}\\tfrac12,i\\plus{}\\tfrac12).$\r\n\r\nThat's why it's open. And it's also closed, since $ \\{i\\}$ is a closed set.",
"Solution_2": "So basically you're saying that for whatever size ball we try to set up around $ i$, that ball still contains $ i$ and nothing else right?",
"Solution_3": "There's a theorem in my book that states:\r\n\r\nIf $ M$ is connected, $ f: M \\rightarrow N$ is continuous, and $ f$ is onto, then $ N$ is connected. The continuous image of a connected is a connected.\r\n\r\nIn writing my proof, can I invoke the theorem above. Then say that because the image of $ f$ has to be connected, and that $ \\mathbb{Z}$ is a disjoint union of clopen proper singleton sets (defn. of disconnected), then $ f$ has to have a singleton set as an image, and in our set $ \\mathbb{Z}$ - a constant.\r\n\r\nDo I have to be more specific, or can this suffice?"
}
{
"Tag": [
"function",
"Asymptote",
"calculus",
"calculus computations"
],
"Problem": "For any real $a$ find the number of real roots for the equation $x=\\tanh ax$",
"Solution_1": "Write the equation as $\\frac{t}{a}=\\tanh t$. Since $t=0$ is always a root, it suffices to look at $t>0$. Two important oservations are: (1) $\\tanh$ is a concave function with a horizontal asymptote; and (2) $(\\tanh t)'=1$ at $t=0$."
}
{
"Tag": [],
"Problem": "Ben left at 10:30 from Toronto and arrived in Sudbury at 2:04. He drove from Toronto to Parry Sound on a 4 lane highway at 100km/hr. Then from Parry Sound to Sudbury he drove on a 2 lane highway at 90km/hr. If the distance from Toronto to Parry Sound is 335 km, at what time did he switch from the 4 lane to 2 lane highway?",
"Solution_1": "Total time is 3 hours, 34 minutes. $ t\\equal{}\\frac{d}{v}\\Rightarrow 90D\\plus{}100d\\equal{}3.5\\bar{6}$ and $ D\\plus{}d\\equal{}335$ Now we have two equations and two unknowns. Once we solve for $ d$, we have $ 100d\\equal{}t$ so we can get the amount of time to get to the 2-lane highway and just add it to 10:30."
}
{
"Tag": [
"combinatorics unsolved",
"combinatorics"
],
"Problem": "is this a planar graph? If yes, how does its dual graph look like?\r\n[img]http://www.mathlinks.ro/Forum/album_pic.php?pic_id=266[/img]",
"Solution_1": "It is planar and the dual graph has vertices A, B, C and edges AB, AB, BC, BC, AC, AC. If you want to know how to construct it, look at http://mathworld.wolfram.com/DualGraph.html"
}
{
"Tag": [
"calculus",
"integration",
"trigonometry",
"logarithms",
"real analysis",
"real analysis unsolved"
],
"Problem": "can you integrate the following ? :D \r\n\r\n$ \\textbf{(i)}\\;\\;\\;\\int_0^1\\;\\;\\left(\\dfrac{\\sin^{\\minus{}1}\\,x}{x}\\right)^2\\;\\;dx$\r\n\r\n$ \\textbf{(ii)}\\;\\;\\;\\int_0^1\\;\\;\\dfrac{\\sin^{\\minus{}1}\\,x}{\\sqrt{\\;1\\plus{}x^2\\;}}\\;\\;dx$",
"Solution_1": "hello, for $ (i)$ it is 4Catalan's constant -$ \\frac{1}{4}\\pi^2$.\r\nSonnhard.",
"Solution_2": "After substitution $ x\\equal{}\\sin t$ the integral becomes $ \\int_0^{\\frac{\\pi}{2}} \\left(\\frac{t}{\\sin t}\\right)^2 \\, \\cos t \\; dt$\r\n\r\nAnd this is $ \\int_0^{\\frac{\\pi}{2}} t^2 \\; \\left(\\frac{\\minus{}1}{\\sin t}\\right)' \\, dt\\equal{}\\left[\\frac{\\minus{}t^2}{\\sin t}\\right]_0^{\\frac{\\pi}{2}}\\plus{}\\int_0^{\\frac{\\pi}{2}} \\frac{2t}{\\sin t} \\, dt\\equal{}\\minus{}\\frac{\\pi^2}{4}\\plus{}4\\, \\underbrace{\\int_0^{\\frac{\\pi}{4}} \\frac{2t}{\\sin 2t} \\, dt}_{\\equal{}J}$\r\n\r\nWhereas $ J\\equal{}\\int_0^{\\frac{\\pi}{4}} \\frac{t}{\\sin t \\; \\cos t} \\, dt\\equal{}\\minus{}\\int_0^{\\frac{\\pi}{4}} t \\, \\frac{1}{\\cot t} \\, \\frac{\\minus{}1}{\\sin^2 t} \\, dt\r\n\\equal{}\\underbrace{\\Big[\\minus{}t \\, \\ln\\cot t \\Big]_0^{\\frac{\\pi}{4}}}_{\\equal{}0}\\plus{}\\int_0^{\\frac{\\pi}{4}} \\ln\\cot t \\; dt$\r\n\r\n$ \\equal{}\\int_0^1 \\frac{\\minus{}\\ln u}{1\\plus{}u^2} \\, du\\equal{}\\underbrace{\\Big[\\minus{}\\ln u \\; \\arctan u \\Big]_0^1}_{\\equal{}0}\\plus{}\\int_0^1 \\frac{\\arctan u}{u} \\, du\\equal{}G$ , hence $ I\\equal{}4\\, G\\minus{}\\frac{\\pi^2}{4}$ .",
"Solution_3": "After substitution $ x\\equal{}\\sin t$ the second integral $ I$ becomes $ \\int_0^{\\frac{\\pi}{2}} \\frac{t}{\\sqrt{1\\plus{}\\sin^2 t}}\\, \\cos t \\, dt$ .\r\n\r\nAnd this is $ \\int_0^{\\frac{\\pi}{2}} t \\, \\left[\\text{arsinh}(\\sin t)\\right]' \\, dt\\equal{}\\underbrace{\\left[t\\, \\text{arsinh}(\\sin t)\\right]_0^{\\frac{\\pi}{2}}}_{\\equal{}\\frac{\\pi}{2} \\, \\text{arsinh} 1}\r\n\\minus{}\\underbrace{\\int_0^{\\frac{\\pi}{2}} \\text{arsinh}(\\sin t)\\, dt}_{\\equal{}J}$ .\r\n\r\nBecause of $ \\text{arsinh} z\\equal{}\\sum_{n\\equal{}0}^\\infty {\\minus{}\\frac12 \\choose n} \\frac{z^{2n\\plus{}1}}{2n\\plus{}1}\\equal{}\\sum_{n\\equal{}0}^\\infty (\\minus{}1)^n \\, \\frac{1}{2^{2n}} \\, {2n\\choose n} \\, \\frac{z^{2n\\plus{}1}}{2n\\plus{}1}$\r\n\r\n$ J\\equal{}\\sum_{n\\equal{}0}^\\infty (\\minus{}1)^n \\frac{1}{2^{2n}} {2n\\choose n} \\frac{1}{2n\\plus{}1} \\int_0^{\\frac{\\pi}{2}} \\sin^{2n\\plus{}1} t \\; dt$, whereas $ \\int_0^{\\frac{\\pi}{2}} \\sin^{2n\\plus{}1} t\\; dt\\equal{}\\frac{1}{2n\\plus{}1} \\left[\\frac{1}{2^{2n}} {2n\\choose n}\\right]^{\\minus{}1}$ .\r\n\r\nTherefore $ J\\equal{}\\sum_{n\\equal{}0}^\\infty (\\minus{}1)^n \\frac{1}{(2n\\plus{}1)^2}\\equal{}G$ , and hence $ I\\equal{}\\frac{\\pi}{2} \\, \\text{arsinh}\\, 1\\minus{}G$."
}
{
"Tag": [
"inequalities",
"function",
"logarithms",
"trigonometry",
"geometry solved",
"geometry"
],
"Problem": "How to prove that: In any triangle $\\triangle ABC$, \\[8 \\omega ^3 \\le ABC\\], where $\\omega$ is the Brocard angle?",
"Solution_1": "Nice problem, in $Prasolov's$ book you can find a nice solution using Jensen(and a couple of other inequalities with Brocard's angle)!",
"Solution_2": "Do you mind to post the solution here? I don't have this book... Thanks so much. :)",
"Solution_3": "[quote=\"yptsoi\"]How to prove that: In any triangle $\\triangle ABC$, \\[8 \\omega ^3 < ABC\\], where $\\omega$ is the Brocard angle?[/quote]\r\n\r\nYptsoi, I guess you meant the inequality to be $8\\omega^3\\leq ABC$ (the sign is $\\leq$, and equality is obtained for equilateral triangles ABC).\r\n\r\nIn fact, this is the famous [url=http://mathworld.wolfram.com/YffConjecture.html][i]Yff inequality[/i][/url]. Here is the proof as given in Prasolov's book:\r\n\r\nConsider the function $f\\left(x\\right)=\\ln\\frac{x}{\\sin x}=\\ln x-\\ln\\sin x$. The functions\r\n\r\n$f^{\\prime}\\left(x\\right)=\\frac{1}{x}-\\cot x$ and $f^{\\prime\\prime}\\left(x\\right)=\\frac{1}{\\sin^2 x}-\\frac{1}{x^2}$\r\n\r\nare both easily seen to be positive for $0 by angle chasing $ \\angle ABB_1\\equal{}\\angle B_1A_1F$ and $ \\angle CBB_1\\equal{}\\angle C_1B_1B\\equal{}\\angle C_1AG\\equal{}\\angle AA_1B_1$ hence result.\r\n2. ABC=AA1F => again by angle chasing we prove that AB1GC1 is cyclic (prove is similar to 1 so I leave it to you)."
}
{
"Tag": [
"Pascal\\u0027s Triangle",
"geometric sequence"
],
"Problem": "I've been looking at pascal's triangle and the binomial expansion theorem and all that kind of stuff, and I've been trying to figure out if something works. But I got stuck with this expression-\r\n\r\ny:^3:/x:^2: - y:^4:/x:^3: + y:^5:/x:^4: - y^6/x:^5: + .... \r\n\r\n- because I don't think I know how to solve things like this. Except I might, I got an answer, but it isn't the one I hoped I would get. So if I\"m right, then I'm wrong, but if I'm wrong then I might be right :lol: if that makes any sense. So is this simplifiable to a non \"...-ing\" expression?",
"Solution_1": "Think geometric progression.\r\n\r\na + ar + ar^2 + ar^3... = a/(1-r) as long as |r| < 1.",
"Solution_2": "[quote=\"TripleM\"]Think geometric progression.\n\na + ar + ar^2 + ar^3... = a/(1-r) as long as |r| < 1.[/quote]\r\n\r\nIt's indeed an Infinite geometric progression with a = y^3/x^2\r\n\r\nand r= -y/x.\r\n\r\nThe series can be summed only if |-y/x| <1\r\n\r\ni.e if |y| < |x| and x !=0\r\n\r\nunder the above restriction, the sum of the series , as stated by TripleM is\r\n\r\ny^3/x^2/(1 + y/x) = y^3/x(x+y)"
}
{
"Tag": [
"puzzles"
],
"Problem": "There are four grades A, B, C, D. A is the best and D is the worst. Four students Jim, Jane, Peter and Lilian appeared in an examination. After the results were published , they made the following statements to a common acquaintance:\r\n\r\n Jim\u2019s Statement: : If I have got A then Lilian has got D. \r\n\r\n Lilian\u2019s Statement: : (i) If I got C then Jim has got D; (ii) Jim\u2019s grade is better than Peter\u2019s grade. \r\n\r\nJane\u2019s Statement: If I didn\u2019t get A then Jim\u2019s grade is not A. \r\n\r\nPeter\u2019s Statement : If Jim\u2019s grade is A, then Jane\u2019s grade is not B, Lilian \u2019s grade is C, but my grade is neither A nor B.\r\n\r\nCan you help the acquaintance to determine the grades of each of the four students, given that all the above statements are true?",
"Solution_1": "I think there are a lot of possibilities.",
"Solution_2": "A possibility is\r\n\r\nJim:C\r\nJane:A\r\nLillian:B\r\nPeter:D",
"Solution_3": "[hide]If Jim has an A, then Jane does not have an A. By Jane's statement, this means that Jim doesn't have an A, which leads to contradiction. This means that Jim does not have an A.\n\nBecause of Lilian's second statement, Jim doesn't have a D.\n\nThus, Jim can have either a B or a C.\n\nLet's assume that Jim has a C. This means that Peter has a D. Jane and Lilian got the A and B. Thus, two solutions are:\nJim: C\nLilian: A\nJane: B\nPeter: D\n\nJim: C\nLilian: B\nJane: A\nPeter: D\n\nLet's assume that Jim has a B. This means Peter can have a C or a D. Let's assume that Peter has a C. This means that Lilian and Jane have the A and D. Thus, here are two more solutions.\n\nJim: B\nLilian: A\nJane: D\nPeter: C\n\nJim: B\nLilian: D\nJane: A\nPeter: C\n\nLet's assume that Peter has a D. This means that Lilian and Jane have the A and C. If Jane has the C, then Jim has the D, which is a contradiction. So, the final solution is:\n\nJim: B\nLilian: A\nJane: C\nPeter: D\n\nThere are 5 solutions altogether.[/hide]",
"Solution_4": "OR...\r\n\r\n[hide]They all are enemies and they lie about their grades. :rotfl: [/hide]",
"Solution_5": "[quote=\"K Sengupta\"]There are four grades A, B, C, D. A is the best and D is the worst. Four students Jim, Jane, Peter and Lilian appeared in an examination. After the results were published , they made the following statements to a common acquaintance:\n\n Jim\u2019s Statement: : If I have got A then Lilian has got D. \n\n Lilian\u2019s Statement: : (i) If I got C then Jim has got D; (ii) Jim\u2019s grade is better than Peter\u2019s grade. \n\nJane\u2019s Statement: If I didn\u2019t get A then Jim\u2019s grade is not A. \n\nPeter\u2019s Statement : If Jim\u2019s grade is A, then Jane\u2019s grade is not B, Lilian \u2019s grade is C, but my grade is neither A nor B.\n\nCan you help the acquaintance to determine the grades of each of the four students, given that all the above statements are true?[/quote] they can't all be true.\r\njim says that if he gets an A then lilian gets a d and peter says that if jim gets an a then lilian got a c they're contidicting eachother..."
}
{
"Tag": [],
"Problem": "A slug climbs ten inches in ten minutes. It then rests two minutes. It continues climbing at a constant rate and rests for two minutes after climbing ten minutes. How many minutes will it take the slug to reach the top of a twenty-foot tower?",
"Solution_1": "20 ft. = 240 inches. Therefore, it will take the slug 24 periods of climbing, which means 23 breaks in between, making it $ 240\\plus{}46\\equal{}\\boxed{286}$"
}
{
"Tag": [
"trigonometry"
],
"Problem": "simplify: ((sin x)^4+4((cos x)^2))^.5 +((cos x)^4 + 4((sin x)^2))^.5 \r\n(i dont have the answer)",
"Solution_1": "Well it can be writen as $ \\sin^{4}{x}\\plus{}\\cos^{4}{x}\\plus{}4(\\sin{x}\\plus{}\\cos{x})$. I'm not sure how much farther though.",
"Solution_2": "It might be helpful to also post the entire problem as well..",
"Solution_3": "$ \\sqrt{\\sin^4 x \\plus{} 4 \\cos^2 x} \\plus{} \\sqrt{\\cos^4 x \\plus{} 4 \\sin^2 x}$\r\n$ \\equal{} \\sqrt{\\sin^4 x \\plus{} 4(1 \\minus{} \\sin^2 x)} \\plus{} \\sqrt{\\cos^4 x \\plus{} 4(1 \\minus{} \\cos^2 x)}$\r\n$ \\equal{} \\sqrt{(\\sin^2 x)^2 \\minus{} 4 \\sin^2 x \\plus{} 4} \\plus{} \\sqrt{(\\cos^2 x)^2 \\minus{} 4 \\cos^2 x \\plus{} 4}$\r\n$ \\equal{} \\sqrt{(\\sin^2 x \\minus{} 2)^2} \\plus{} \\sqrt{(\\cos^2 x \\minus{} 2)^2}$\r\n$ \\equal{} |\\sin^2 x \\minus{} 2| \\plus{} |\\cos^2 x \\minus{} 2|$\r\n$ \\equal{} 2 \\minus{} \\sin^2 x \\plus{} 2 \\minus{} \\cos^2 x$\r\n$ \\equal{} 4 \\minus{} (\\sin^2 x \\plus{} \\cos^2 x)$\r\n$ \\equal{} 4 \\minus{} 1$\r\n$ \\equal{} 3$.\r\n\r\nNote that for all real $ x$, $ |\\sin x| \\le 1$ and $ |\\cos x| \\le 1$, from which it follows that $ |\\sin^2 x \\minus{} 2| \\equal{} 2 \\minus{} \\sin^2 x$, and so forth."
}
{
"Tag": [],
"Problem": "$ a) \\frac {6.19}{6.24 \\minus{} 3.12\\times 2}$\r\n\r\n$ b) \\frac {7.8}{ \\minus{} 5.64 \\minus{} 3.1233}$\r\n\r\n$ c) \\frac {2.4\\div 3}{0.6 \\minus{} 1.8\\div 3}$\r\n\r\n$ c) \\frac {3.4\\times 1.4}{ \\minus{} 1.8 \\minus{} 3\\frac {2}{3}\\times ( \\minus{} 2)}$\r\n\r\n :wink: No calculator !",
"Solution_1": "This is just a matter of plugging it into your calculator, I'm guessing.\r\n\r\n[hide=\"answers\"]\na) undefined, because $ 6.24\\minus{}3.12\\cdot2\\equal{}0$\nb)$ \\minus{}.8901$\nc) again, undefined\nd) $ .7843$[/hide]",
"Solution_2": "I thought it would be a good idea to bring up why A and C are undefined. You can [b]never[/b] divide a number by 0 because multiplication and divsion are interchangable meaning that the product of the divisor and quotient will equal the dividend, and in mutliplication of 2 numbers the product divided by one of te number will always equal the other number.",
"Solution_3": "SOLUTION OF a is\r\n6.19/0 =infinity :rotfl:",
"Solution_4": "[quote=\"salahcool\"]SOLUTION OF a is\n6.19/0 =infinity :rotfl:[/quote]\r\nYes. The positive infinity.\r\nSee the endless ascending sequence $ \\frac{1}{2}, \\frac{1}{1}, \\frac{1}{\\frac{1}{2}}, \\frac{1}{\\frac{1}{3}}, \\frac{1}{\\frac{1}{4}}, \\frac{1}{\\frac{1}{10}}, \\frac{1}{\\frac{1}{100}}, \\frac{1}{\\frac{1}{1000}} ... \\frac{1}{0}$\r\nThen fewer is a discriminant then bigger is a fraction."
}
{
"Tag": [
"integration",
"trigonometry",
"calculus",
"calculus computations"
],
"Problem": "Find the volume of the solid enclosed by the cylinder $ {{x}^{2}}\\plus{}{{y}^{2}}\\equal{}9$ and the planes y + z = 5 and z = 1. Help me, please.",
"Solution_1": "$ \\begin{gathered}\r\n z \\plus{} y \\equal{} 5,{\\text{ }}z \\equal{} 1,{\\text{ }}{x^2} \\plus{} {y^2} \\equal{} 9. \\hfill \\\\\r\n D \\equal{} \\left\\{ {\\left. {\\left( {x,y} \\right)} \\right|{\\text{ }} \\minus{} \\sqrt {9 \\minus{} {x^2}} \\leqslant y \\leqslant \\sqrt {9 \\minus{} {x^2}} ,{\\text{ }} \\minus{} 3 \\leqslant x \\leqslant 3} \\right\\}. \\hfill \\\\\r\n V \\equal{} \\iint\\limits_D {dxdy}\\int\\limits_1^{5 \\minus{} y} {dz} \\equal{} \\iint\\limits_D {\\left( {4 \\minus{} y} \\right)dxdy} \\equal{} \\int\\limits_{ \\minus{} 3}^3 {dx} \\int\\limits_{ \\minus{} \\sqrt {9 \\minus{} {x^2}} }^{\\sqrt {9 \\minus{} {x^2}} } {\\left( {4 \\minus{} y} \\right)dy} \\equal{} \\hfill \\\\\r\n \\equal{} 2\\int\\limits_0^3 {\\left. {\\left( {4y \\minus{} \\frac{{{y^2}}}\r\n{2}} \\right)} \\right|_{ \\minus{} \\sqrt {9 \\minus{} {x^2}} }^{\\sqrt {9 \\minus{} {x^2}} }dx} \\equal{} 16\\int\\limits_0^3 {\\sqrt {9 \\minus{} {x^2}} dx} \\equal{} \\left\\{ \\begin{gathered}\r\n x \\equal{} 3\\sin t,{\\text{ }}dx \\equal{} 3\\cos tdt, \\hfill \\\\\r\n 0 \\leqslant x \\leqslant 3,{\\text{ }}0 \\leqslant t \\leqslant \\frac{\\pi }\r\n{2} \\hfill \\\\ \r\n\\end{gathered} \\right\\} \\equal{} \\hfill \\\\\r\n \\equal{} 48\\int\\limits_0^{\\pi /2} {\\sqrt {9 \\minus{} 9{{\\sin }^2}t} \\cos tdt} \\equal{} 144\\int\\limits_0^{\\pi /2} {{{\\cos }^2}tdt} \\equal{} 72\\int\\limits_0^{\\pi /2} {\\left( {1 \\plus{} \\cos 2t} \\right)dt} \\equal{} \\hfill \\\\\r\n \\equal{} \\left. {72\\left( {t \\plus{} \\frac{1}\r\n{2}\\sin 2t} \\right)} \\right|_0^{\\pi /2} \\equal{} 36\\pi {\\text{ }}\\left( {{\\text{cubic units}}} \\right). \\hfill \\\\ \r\n\\end{gathered}$"
}
{
"Tag": [
"limit",
"integration",
"inequalities",
"calculus",
"calculus computations"
],
"Problem": "Find $ \\lim_{n\\to\\infty} \\frac{1}{n^2} \\sum_{k\\equal{}1}^n \\sqrt[4]{k(k\\plus{}1)(k\\plus{}2)(k\\plus{}3)}$.",
"Solution_1": "[color=darkred] \\lim_{n\\to\\infty} \\frac{1}{n^2} \\sum_{k=1}^n \\sqrt[4]{k(k+1)(k+2)(k+3)}=\\frac{1}{2}[/color]",
"Solution_2": "[quote=\"shubin\"]$ \\lim_{n\\to\\infty} \\frac{1}{n^2} \\sum_{k\\equal{}1}^n \\sqrt[4]{k(k\\plus{}1)(k\\plus{}2)(k\\plus{}3)}\\equal{}\\frac{1}{2}$[/quote]\r\n\r\nHow did you get it?",
"Solution_3": "[quote=\"kunny\"][quote=\"shubin\"]$ \\lim_{n\\to\\infty} \\frac {1}{n^2} \\sum_{k \\equal{} 1}^n \\sqrt [4]{k(k \\plus{} 1)(k \\plus{} 2)(k \\plus{} 3)} \\equal{} \\frac {1}{2}$[/quote]\n\nHow did you get it?[/quote]\r\n$ \\int\\limits_{0}^{1} \\sqrt[4]{(x^2\\plus{}3x\\plus{}1)^2\\minus{}1}dx \\equal{}\\frac{1}{2}$",
"Solution_4": "[quote=\"Hong Quy\"]$ \\int\\limits_{0}^{1} \\sqrt [4]{(x^2 \\plus{} 3x \\plus{} 1)^2 \\minus{} 1}dx \\equal{} \\frac {1}{2}$[/quote]\r\n\r\n$ \\int_{0}^{1} x \\, dx \\equal{} \\frac {1}{2}$.\r\n\r\nOne may prove like this:\r\n\r\n$ \\frac {1}{n^2} \\sum_{k \\equal{} 1}^n \\sqrt [4]{k(k \\plus{} 1)(k \\plus{} 2)(k \\plus{} 3)}$ $ \\equal{} \\frac {1}{16} \\sum_{k \\equal{} 1}^n \\sqrt [4]{\\left(\\frac {4k}{n}\\right)\\left(\\frac {4k \\plus{} 1}{n}\\right)\\left(\\frac {4k \\plus{} 2}{n}\\right)\\left(\\frac {4k \\plus{} 3}{n}\\right)} \\cdot \\frac {4}{n}$ $ \\equal{} \\frac {1}{16} \\sum_{k \\equal{} 1}^n \\frac {4k \\plus{} c_k}{n} \\cdot \\frac {4}{n}$\r\n\r\nfor some $ 0 < c_k < 3$, by intermediate value theorem applied to the inequality\r\n\r\n$ \\frac {4k}{n} < \\sqrt [4]{\\left(\\frac {4k}{n}\\right)\\left(\\frac {4k \\plus{} 1}{n}\\right)\\left(\\frac {4k \\plus{} 2}{n}\\right)\\left(\\frac {4k \\plus{} 3}{n}\\right)} < \\frac {4k \\plus{} 3}{n}.$\r\n\r\nThen it converges to $ \\frac {1}{16}\\int_{0}^{4} x \\, dx \\equal{} \\int_{0}^{1} t \\, dt$."
}
{
"Tag": [
"induction",
"graph theory",
"combinatorics proposed",
"combinatorics"
],
"Problem": "There is a complete Graph $K_n$ and all its edges are colored in two colors, prove there exists a Hamilton Cycle such that going arround it one only change of color once.",
"Solution_1": "Ok, I hope it's ok.....\r\nSo, just do it by induction...I mean, consider a bicolored $K_{n+1}$. Take some $K_n$ of it, apply induction, and the vertex left is $A$. If the cycle is of the form $A-X_1-B-X_2-C$, where all the edges in the $A$ and $C$ chains are of the same color, and $X_i$ are \"intermediate\" vertices. Now just consider the color of $A-X_i$ and look to the left or right of it.....\r\nI hope I didn't miss any cases..... :oops:"
}
{
"Tag": [
"geometry",
"parallelogram",
"circumcircle",
"analytic geometry",
"function"
],
"Problem": "Does anyone know any general theorems that give you information as to how a line can bisect the area of a polygon? I think I have seen such a thm. before, but I thought it was too difficult to learn at the time.",
"Solution_1": "I found one. Can anyone find the proof?\r\nIf a line bisects the area of a parallelogram, then the line passes through the intersection of the diagonals of the parallelogram.",
"Solution_2": "if it is a regular polygon, a line through the circumcenter suffices, one exercise related exercise that you should work on is: prove that such a line exists\r\n[hide=\"hint\"]continuity[/hide]",
"Solution_3": "I'll followed your hint Altheman. Tell me if you see anything incorrect.\r\nThm: There exists atleast one line that bisects the area of a concave polygon.\r\nProof: Consider a concave polygon $A$ and an arbitrary line $l_k$ situated in the coordinate plane. Furthermore, let $l_k$ be defined by the equation $y=mx+k$ where $m$ is a constant. Finally, note that $A$ and $l_k$ are infinite sets of pts.\r\nNow define $A(k)$ to equal the lower area bounded by $A$ and $l_k$. We see that, for different values of k, $A(k)$ ranges from $0$ to $A$'s entire area. It is also easy to see that $A(k)$ is a continuous function of $k$ since $A$ is concave.\r\nThus, if we let $k_0$ be the value of $k$ that makes $A(k)=0$, and $k_t$ be the value of $k$ that makes $A(k)$ equal the total area of $A$, then there exists some value $k'$ such that $k_0\r\n 1= x(1)^s(k) +x(2)^s(k) +.....+x(k)^s(k) \r\n \\leq x(1)^s(k+1) +x(2)^s(k+1)+.......+x(k)^s(k+1)\r\n < x(1)^s(k+1)+x(2)^s(k+1)+.........+x(k)^s(k+1) +x(k+1)^s(k+1) =1\r\n Nonsense.\r\n --> x(k+1)>s(k) for all k\r\n \r\n2)\r\nFor all k , s(k) \\leq s\r\n {If s(k)>s ,we deduce \r\n 1= x(1)^s(k) +x(2)^s(k) +.....+x(k)^s(k) \r\n \\leq x(1)^s+x(2)^s+.........+x(k)^s +x(k+1)^s\r\n oo}[x(n)^c]=lim sum{n-->oo}[x(n)^s] \r\n So \r\n lim sum{n-->oo}[x(n)^c-x(n)^s] =0 (*)\r\n But , for 0